Tieteenfilosofiaa: korrelaatiot vs kausaliteetti - reduktionismi vs systeemit

Anonyymi-ap

Erilaiset systeemitasot karkeasti määriteltynä:

alkeishiukkaset-atomit-molekyylit-eliöt-eliöiden yhteisöt

Erilaisten systeemien komponenttien käyttäytyminen johtuu siitä minkälaisen ja minkätasoisen systeemin osia ne ovat. Eliöiden tasolla on aineenvaihduntaa mikä tarkoittaa että niiden kehojen atomien ja molekyylien taso vaihtuu jatkuvasti vaikka kehon komponentien rakenne ja toiminta pysyy samana.

Korrelaatiot sotketaan usein kausaliteettiin koska ei tiedosteta erilaisia systeemitasoja (esim. neurokorrelaatit ja dna-korrelaatit joissa systeemin kokonaisuus määrittelee sen systeemin käyttäytymistä)

Esim. ihmisten yhteiskunnan tasolla lainsäädäntö ja erilaiset sopimukset määrittelevät miten yksilöiden enemmistö tyypillisesti käyttäytyy siinä systeemissä eli yhteisössä.

Aivojen neuronit ja geenit eivät määrittele esim. junien aikatauluja vaikka ne ovatkin osia erilaisten kokonaisuuksien komentoketjuissa joissa kausaliteettien aloittajana on joku yhteiskunnan päättävä taso ja sen tahtotila.

Alkeishiukkasilla ei ole suurta merkitystä erilaisten kemiallisten yhdisteiden ja sitä monimutkaisempien systeemien käyttäytymisessä vaikka niiden käyttäytyminen tietysti korreloi sen kokonaisuuden mukana missä ne kulloinkin vaikuttavat.

Syteemitason kausaliteetti on tilastollista eikä koskaan 100% tarkkuudella ennustettavissa vaikka sen systeemien komponenttien ja niiden osajärjestelmien tasolla voidaan lähestyä jonkinlaista determinismiä.

Tästä voi päätellä että fysiikka on tieteenä melko yliarvostettua koska se määrittelee ainoastaan reunaehtoja sille millaista käyttäytymistä erilaisissa fysiikalisista kompontenteista koostuneissa systeemeissä voi olla eikä tuota kausaliteetteja vaan lähinnä vain tilastollisia korrelaatioita erilaisten systeemikokonaisuuksien kannalta.

Todellisuus on paljon monimuotoisempi ja monitasoisempi kuin pelkästään matemaattis-luonnontieteellisen hahmotuksen kautta voi näyttää.

30

478

    Vastaukset

    Anonyymi (Kirjaudu / Rekisteröidy)
    5000
    • Anonyymi

      Kvanttimaailmasta pystyy johtamaan kaikki ylemmät tasot kuten junien aikataulut.

      Aina kun mennään ylemmälle tasolle tapahtuu alemman tason karkeistus ja informaatiota menetetään.

      Tämä on provo.

      • Anonyymi
        UUSI

        😍😍😍😋😋😋😋😍😍😍

        💋 ­­N­­y­­m­­f­­­o­­­m­­­a­­­a­n­­­i -> https://ye.pe/finngirl21#18053366L

        🔞❤️💋❤️💋❤️🔞❤️💋❤️💋❤️🔞


    • Anonyymi

      Ihmiset ovat korkean entropian tilassa, kun ajattelee informaatioteriassa, että niiden tekoja tai liikettä ei voi tietää. Haudoissa ihmiset eivät liiku ja elävät matalimman entropian tilassa. Taivaalla parvissa lentävät ihmiset ovat jotain ihmeellistä, koska ovat näiden kahden välimaastosta. Sanot varmaan että ihmisten asiat eivät ole näin, koska pienten solujen taso on ainoa taso, millä on väliä.

      Kyseessä ei ole termodynaaminen entropia, josta seuraisi jotain homogenisoitumista, ainakaan jos ei ole sanonut sellaista näkevänsä. Periaatteessa ihmisillä voisi olla jokin raja, miten paljon ihmisiä pitää olla samassa alueessa, jotta ne näyttäisivät siltä, että ottamalla esim. yhden valtion sisällä olevat ihmiset, tämä valtio on ensinäkin identtinen muiden valtioiden kanssa, ja toiseksi tämä johtuu siitä, että näyte muodostaisi sellaisen kokoelman ihmisiä, että siinä kokelmassa on niin monta ihmistä, että siinä on edustettu juuri silloin kaikki ihmisen tekemisen mahdollisuudet (jollain reunaehdolla, missä mm. sen ajan tiede päättää voiko ajaa lentävällä autolla). Ja vielä moneen kertaan siten, että esim. Michelangeloita tekemässä Daavidin patsasta on siinä näytteessä miljoona. Ja Daavideja voittamassa Goljateja on miljardi, ja Goljateja voittamassa ainakin kaksi. Ihmismäärä, joka tarvitaan siihen, että ihmisiä on mahdoton ennustaa on kuitenkin paljon pienempi kuin tämä, ja entropia on merkittävä käsite jo ennen tätä.

      "--Korrelaatiot sotketaan usein kausaliteettiin koska ei tiedosteta erilaisia systeemitasoja (esim. neurokorrelaatit ja dna-korrelaatit joissa systeemin kokonaisuus määrittelee sen systeemin käyttäytymistä).
      ...
      Alkeishiukkasilla ei ole suurta merkitystä erilaisten kemiallisten yhdisteiden ja sitä monimutkaisempien systeemien käyttäytymisessä vaikka niiden käyttäytyminen tietysti korreloi sen kokonaisuuden mukana missä ne kulloinkin vaikuttavat.--"

      Onko Sotkija-A sanonut, että on olemassa korrelaatio? Mikä korrelaatio on kyseessä, kun puhut korrelaatiosta, jossa mainitaan 'kokonaisuuden mukana' korrelointi? Eikö kokonaisuus siinä tarkoita juuri toista systeemin tasoa, vai onko se vain kielivirhe ja oikeasti tarkoitat korrelaatiota moneen muuhun alkeishiukkaseen? Jos kausaliteetista tehdään alkeishiukaksen korrelaatio toisiin alkeishiukkasiin, niin ei tarvitse ymmärtää mitään mistään toisesta tasosta. Eikä minkään sotkun syynä olla tekemättä tällä tavalla voi olla se, että olisi taso jäänyt. Sanasta 'dna-korrelaatti', jonka olet keksinyt itse, ei saa mitään selvää, mikä systeemi on, ja onko tasoja monta. Jos sotkija-A on sanonut ettei korrelaatiota ole, niin todista jonkun korrelaation olemassaolo, koska ilmeisesti A:n kausaliteetti ei tee asioiden välille korrelointeja. Lisäksi kausaatiot voisi osoittaa olemattomaksi ilman, että sen tilalle tulee korrelaatio, ja tällaiset kausaatioiden kumoukset voisivat olla esityksenä vaikuttavampia kuin mikään muu teksti.

      Kun sanotaan, että dna:ta ei ole olemassa ja on vain alkeishiukkasten joukko, tämä ei perustu kausaliteettiin. Mihinkään niistä, koska ainakin lisäksi tarvitaan jokin muu kausaliteetin käsite kuin signaalien nopeus -kausaliteetti. On sinun huonoa tasojen ymmärrystäsi ajatella, että joku olisi sanonut tasojen liittyvän kausaaliteettiin. Jos joku sanoo esim. että tupakointi aiheuttaa syöpää, tässä ei ole kyseessä väite, jossa tupakointi on alkeishiukkasista koostuvaa kemiallista ainetta (johtuen siitä, että tupakan savu laatikossa tottelee yhtälöitä eikä koskaan keskustele kanssamme), ja että syöpä on syövän saanut ihminen (koska syöpä tappaa ja lopettaa keskustelumme heidän kanssaan). Oikeat tasot voivat olla täysin mielivaltaiset mutta samat tasot riippuen siitä, mitä asiasta on sanottu. Alkeishiukkasten taso vaatii paljon muutakin selitystä, kuin että käsin allekirjoitetuista kuiteista näkyvä tupakanostotoiminta on johtanut kuolemiin nuoremmalla iällä.

      Tosi usein jotta voisi väittää asioita kaikilla tasoilla, tarvitaan kyseisen tason osoitus asiasta, koska jos kuitteja ei olisi laskettu ja nähty vain yksi keuhkosolun rappeuma, niin ei olisi ollut oikein uutisoida kuolemaa koko kansalle. Alkeishiukkastaso on suuressa eliössä sellainen, että koko lopputulosta ei näe, koska nähty alkeishiukkastapahtuman seuraus voi korjaantua toisella alkeishiukkastapahtumalla, jota ei vielä nähnyt savun seasta.

    • Anonyymi

      Aika tärkeä tieto?

      Tekoäly:

      ( joka voi olla myös väärässä)

      mikä on Quantum gate?

      Quantum gate on kvanttitietokoneissa käytetty operaatio, joka muuttaa kvanttibittien tiloja. Se vastaa klassisten tietokoneiden loogisia porteja, kuten AND, OR ja NOT -porteja. Quantum gateilla voidaan suorittaa kvanttioperaatioita, kuten kvanttiohjelmoinnin, kvantti-informaation koodauksen ja kvanttialgoritmien toteuttamisen avulla. Ne ovat olennainen osa kvanttilaskentaa ja mahdollistavat kvanttibittien monimutkaisten tilojen manipuloinnin.

      ...

      • Anonyymi

        Oljkos se Mary Quant yks näestä vanttiteoriaan perustajista?


    • Anonyymi

      Tekoäly kehittyy vitsinkertomisessa koko ajan:

      Kuinka monta kvanttimekaniikan tutkijaa tarvitaan vaihtamaan hehkulamppu moni maailmassa?
      Vain yksi, mutta samanaikaisesti myös loputtomasti muita versioita, jotka tekevät saman asian samanaikaisesti!

      • Anonyymi

        "Filosofialla ei ole mitään annettavaa oikealle tieteelle."

        ERI PALSTOILLA JOKU "TIEDEMIES" JATKUVASTI HOKEE SELLAISTA.

        Et siis tajua, että mitä enemmän tiedät, sitä vähemmän ymmärrät tietäväsi?
        Kenties et ole mikään tiedemies, jos sellaista yksinkertaista asiaa et ymmärrä.


        Kuinka paljon pystymme ajattelemaan? Mikä on? Mikä ei ole? Miten meidän on mahdollista tietää asioiden luonne? Newton sanoi: Newton sanoi: "Kerään kiviä rannalta. Tiedon valtameri levittäytyy edessäni, ja minä vain kerään kiviä sen rannalta." Mitä todellisuudessa on olemassa? Mikä on mitä? Kaikki on achintya (käsittämätöntä).

        Newton tajusi sen, mutta eri palstoilla muita halveksiva "tiedemies" ei tajuaa sitä.


      • Anonyymi
        Anonyymi kirjoitti:

        "Filosofialla ei ole mitään annettavaa oikealle tieteelle."

        ERI PALSTOILLA JOKU "TIEDEMIES" JATKUVASTI HOKEE SELLAISTA.

        Et siis tajua, että mitä enemmän tiedät, sitä vähemmän ymmärrät tietäväsi?
        Kenties et ole mikään tiedemies, jos sellaista yksinkertaista asiaa et ymmärrä.


        Kuinka paljon pystymme ajattelemaan? Mikä on? Mikä ei ole? Miten meidän on mahdollista tietää asioiden luonne? Newton sanoi: Newton sanoi: "Kerään kiviä rannalta. Tiedon valtameri levittäytyy edessäni, ja minä vain kerään kiviä sen rannalta." Mitä todellisuudessa on olemassa? Mikä on mitä? Kaikki on achintya (käsittämätöntä).

        Newton tajusi sen, mutta eri palstoilla muita halveksiva "tiedemies" ei tajuaa sitä.

        Sen kun hokee, tyhmiä on aina ollut.


      • Anonyymi
        Anonyymi kirjoitti:

        Sen kun hokee, tyhmiä on aina ollut.

        Niin on.


      • Anonyymi
        Anonyymi kirjoitti:

        Sen kun hokee, tyhmiä on aina ollut.

        Sanoppas muuta.


      • Anonyymi
        Anonyymi kirjoitti:

        Niin on.

        joopajoo


    • Anonyymi

      Tekoäly:

      Kerro lisää ajatuksesta jonka mukaan säieteoria linkittää materian avaruuteen

      Säieteoria on teoreettinen fysiikan haara, joka pyrkii yhdistämään yleisen suhteellisuusteorian ja kvanttimekaniikan. Se ehdottaa, että perustavanlaatuiset hiukkaset eivät ole pisteitä, vaan pieniä värähteleviä säikeitä tai silmukoita, joilla on tiettyjä ominaisuuksia.

      Säieteoria tarjoaa myös mahdollisuuden ymmärtää, miten materia ja avaruus liittyvät toisiinsa. Sen mukaan avaruus ei ole vain tyhjä tausta, vaan se on itse asiassa jännitteisten säikeiden verkosto, joka muodostaa avaruuden rakenteen. Nämä säikeet voivat kiertyä ja solmita toisiinsa luoden monimutkaisia kudoksia.

      Materia puolestaan koostuu säikeiden erilaisista värähtelytiloista ja vuorovaikutuksista. Erilaiset hiukkaset, kuten elektronit ja kvarkit, voivat syntyä ja vuorovaikuttaa säieteorian puitteissa. Näiden säikeiden värähtelevien tilojen ja vuorovaikutusten monimutkaisuus ja monimuotoisuus vaikuttavat siihen, miten materia ilmenee ja miten se vuorovaikuttaa avaruuden kanssa.

      Yhteenvetona voidaan sanoa, että säieteoria tarjoaa puitteet ymmärtää, miten materia ja avaruus linkittyvät toisiinsa. Se kuvaa avaruuden rakenteen säikeiden verkostona ja selittää, miten nämä säikeet muodostavat materian eri ilmenemismuodot ja vuorovaikuttavat keskenään. Kuitenkin on tärkeää huomata, että säieteorian osalta tutkimus ja kehitys ovat edelleen käynnissä, ja se on aktiivisen tutkimuksen kohde fysiikan alalla.

      Mm
      Nn..

      • Anonyymi

        kerro itse,


    • Anonyymi

      Patologian professori Neil Thiesellä on oikeansuuntainen ja mullistava näkemys suhteessa perinteiseen geenikeskeiseen biologiaan ja evoluutioteoriaan ja hän selittää hyvin miksi reduktionistinen mallinnus tieteessä johtaa usein harhapoluille.


      Neil Theise - How Do Complexity and Emergence Create a Cosmos?

      https://www.youtube.com/watch?v=DLRynSjBD9M


      Ihmiskeho on bakteerien ja ihmisen solujen muodostama kokonaisuus jossa on 99% bakteereja ja vain yksi prosentti genomin mukaisia soluja ihan samoin kuin atomaarisen aineen suhde sähköisesti aktiiviseen ja itseorganisoitumiseen kykenevään plasmaan .

      Jos tutkitaan vain mikroskooppisella reduktionistisella tasolla esim. soluja niin ei käytännössä ymmärretä juuri mitään kokonaisuudesta ja fysiikassa on sama tilanne koska ei tajuta että nimenomaan sähköisesti aktiivinen plasma on se olennaisin asia fysiikassa, tähtitieteessä ja kosmologiassa ja koska lähes kaikki huomio on kiinnittynyt vain siihen yhteen prosentiin kokonaisuudesta jonka perusominaisuus on itseorganisoituminen ja kompleksisuus jota kuvaillaan emergenssin käsitteen kautta joka ei sellaisenaan ole mikään kausaalinen selitys (=vastaus 'miksi' kysymykseen) vaan pelkkä kuvaus ja suurin osa luonnontietelijöistä kuvittelee että matemaattinen säännönmukaisuuksien kuvaus olisi myös kausaalinen seliitys.

      Mahdollisia vastauksia "miksi?" kysymykseen voivat olla esim. ns. luonnonlait, sattuma tai elöiden tasolla tahto/intentio. Matemaattiset fysiikan kaavat ovat vain ilmiöiden kuvauksia ja vastauksia "miten?" kysymyksiin josta voi päätellä että tieteen metodi ei edes mahdollista todellisuuden syvempää ymmärtämistä joka edellyttää ihmisen itsensä kehittämistä erityisesti tunteiden tasola sekä toimivan intuition ja intention harjoittelua jota voi kutsua myös laumasta yksilöllistymisen prosessiksi joka on kaikkien ns. henkisten perinteiden päämäärä vaikka se usein sotketaan institutionaaliseen laumauskomiseen kuten perinteisissä uskonnoissa ja myös sekulaarissa skientismin uskonnossa joka pyrkii dogmatisoimaan ja jähmettämään olemassaolevia melko alkeellisia akateemisen tieteen paradigmoja.

      ...
      Hyvä ja käytännönläheinen kuvaus todellisesta ja toimivasta tieteen metodista joka käytännössä kumoaa Popperin falsifioitavuusperiaatteen epäkäytännöllisenä filosofisena haaveiluna joka ei toimi käytännön kokeellisen tieteen tasolla.


      Falsifiability and Messy Science

      https://www.youtube.com/watch?v=xG3-_tgDE0k

      Parhaimmat tulokset tieteessä saadaan nimenomaan vapaamuotoisesti ja intuitiivisesti kokeilemalla ja ideoiden tasolla leikkimillä eikä niinkään minkään kaavamaisen ja systemaattisen muodollisesti tieteen metodin pätevän avulla.

      Luonto on monessa suhteessa paljon nerokkaampi ja luovempi kuin ihminen ja hyvä esimerkki on esim. microbi joka osaa tuottaa kultaa aineenvaihduntansa avulla.

      Gold-digging bacterium makes precious particles

      https://www.nature.com/articles/nature.2013.12352

      Esim. Victor Schauberger oli hyvä esimerkki luonnntieteilijäsytä joka ymmärsi että olennaisinta on yrittää oppia ja matkia luonnonilmiöitä ilman mitään akateemista ehdollistamista joka enimmäkseen on pelkästään turhaa painolastia :D


      Viktor Schauberger - Comprehend and Copy Nature (Documentary of 2008)


      https://www.youtube.com/watch?v=yXPrLGUGZsw

      Lisää aineistoa Schaubergerin ideoista löytyy archive.org sivustolta.




      B

      • Anonyymi

        Paskanjauhannalla ei ole tieteelle mitään annettavaa.


      • Anonyymi

        B:
        "Neil Theise - How Do Complexity and Emergence Create a Cosmos?
        https://www.youtube.com/watch?v=DLRynSjBD9M
        Ihmiskeho on bakteerien ja ihmisen solujen muodostama kokonaisuus jossa on 99% bakteereja ja vain yksi prosentti genomin mukaisia soluja ihan samoin kuin atomaarisen aineen suhde sähköisesti aktiiviseen ja itseorganisoitumiseen kykenevään plasmaan."

        Tässä mainittu reduktionismi ei ole filosofinen käsite vaan ns. reduktionismista luopuminen on täysin käytännöllinen pakko. Kompleksisuusteorian ei ole tarkotus pohtia ontologisia käsitteitä, vaan ratkaista jokin ongelma algoritmisesti. Moni ongelma ratkaistaan käytännön syistä karkeasti karkealla tasolla ottamatta pieniä osia huomioon.

        Jos on olemassa ihminen, mutta hänen terveyttään ja elämäänsä ei ratkaista käsien heilutuksen tasolla, vaan soluissa ja bakteereissa, tämä kertoo, että on olemassa paljon pienempiä skaaloja, mistä kompleksisuusteoria voi valita jonkin tason, koska sen on kuitenkin pakko valita jotain muuta kuin kädet, jotta olisi jokin, mikä tulee olemaan sopiva sen oletetun ongelman ratkaisemiseen.

        Kaikki Theisen reduktion vastaiset lauseet ovat vain kielikukkasia. Hänen oikea tarkoituksensa on odottaa tulevaisuudessa kaikkia hyödyllisiä matemaattisia malleja biologian esittämiseksi.

        Theise sanoi, että sinulla ei ole suolistojörjestelmää, elleivät DNA:t koordinoidu ja järjestäydy bakteerien olemassaoloa varten. Tässä lauseessa DNA on molekyylin kokoinen asia - eikä solun kokoinen.

        Vanha tiede ns. pysähtyi solun kohdalla, koska solua ei voi ns. jakaa tiettyihin asioihin, mitkä näkyvät pikemminkin solua suuremmalla tasolla. Tämä tarkoittaa sitä, että solu rikkoi skaalainvarianssin. Olisi ollut todella väärin jatkaa ajattelua, että esim. lihas kulkee solun läpi samanlaisena lihasmassana. Tällainen tasainen lihas-kontinuumi (löysän mutta kiinteän aineen yhtälöt oletettuna ilman sellaista hilaa, joka olisi mikroskoopin nähtävissä (*)) olisi ollut kuitenkin teoriassa helppo ratkaista, mutta se on kai tässä vain mielikuvituksekkaasti mainittu, eikä sellaisen esim. tulosta ja jotain hyötyjä ole kukaan nähnyt.

        (*) Seuraavassa lauseesa Theise ilmeisesti leikittelee sillä ajatuksella, että jokin solun sisällä oleva asia olisi voinut palvella tämän hilan osana, jolloin kontinuumiteoria tästä hilasta olisi voinut olla jotain, mihin olisi pyritty vähän aikaa. Periaatteessa kukaan ei estä näkemästä myöskään sellaista hilaa, missä on sekoitettu kristalli sekä solun seinää, että solun tumaa, tai kaikkia solun sisäisiä osia erikseen. Mutta syy miksi konseptilla, missä tämä approksimoidaan myös liikeyhtälöksi ei tee mitään, on se, että todella paljon biologisia asioita tapahtuu, kun solut ja niiden osat solun sisällä liikkuvat toistensa suhteen yksilöinä.

        Mainittu Toshio Yanagita käytty mikroskooppia ja katsoi proteiinimolekyylejä (nimenomaan näitä molekyylejä erotettuna koko muusta ruumiista) tarkaan laservalon alla, ja hänestä tuli/tulee lääketieteen mullistaja uuteen kompleksisuuden ja emergenssin aikaan. (Energiamolekyyli ATP ei saa liikettä aikaan kuten joku biologi luuli, vaan se saa aikaan olemassaolevan liikkeen epäsatunnaistumisen. Epäsatunnaistuminen on kuitenkin siitä peräisin edelleen, että ATP+myosiini vapauttaa energiaa.)
        https://www.ncbi.nlm.nih.gov/pmc/articles/PMC3521640/
        https://www.researchgate.net/publication/244753006_Model_describing_the_biased_Brownian_movement_of_myosin

        'This article describes how biased Brownian movement and the Brownian search-and-catch mechanism are the basis for muscle contraction. Simulations show that these two mechanisms, as evidenced from single molecule measurements, play a critical role in determining the characteristic properties of muscle....
        Further, it has been demonstrated that myosin motors are typical biological molecular machines that attain their function while under the influence of thermal fluctuations.'

        (Tämä on hyvä esimerkki siitä, miten itseorganisaatiossa on usein kyse jostain, missä entropiaa on paljon.)

        Theisen mukaan 'molekyylit ovat itseorganisoituvia atomeja eivätkä mitään muuta'. Samalla tavalla ihminen on itseorganisoituvia atomeja eikä mitään muuta? (Seuraavaksi hän sanoo, että ihmiskehossa tähän tiettyjen osien organisoitumiseen auttaa DNA, mutta käsitys on varmasti suppea totuus.)


      • Anonyymi
        Anonyymi kirjoitti:

        B:
        "Neil Theise - How Do Complexity and Emergence Create a Cosmos?
        https://www.youtube.com/watch?v=DLRynSjBD9M
        Ihmiskeho on bakteerien ja ihmisen solujen muodostama kokonaisuus jossa on 99% bakteereja ja vain yksi prosentti genomin mukaisia soluja ihan samoin kuin atomaarisen aineen suhde sähköisesti aktiiviseen ja itseorganisoitumiseen kykenevään plasmaan."

        Tässä mainittu reduktionismi ei ole filosofinen käsite vaan ns. reduktionismista luopuminen on täysin käytännöllinen pakko. Kompleksisuusteorian ei ole tarkotus pohtia ontologisia käsitteitä, vaan ratkaista jokin ongelma algoritmisesti. Moni ongelma ratkaistaan käytännön syistä karkeasti karkealla tasolla ottamatta pieniä osia huomioon.

        Jos on olemassa ihminen, mutta hänen terveyttään ja elämäänsä ei ratkaista käsien heilutuksen tasolla, vaan soluissa ja bakteereissa, tämä kertoo, että on olemassa paljon pienempiä skaaloja, mistä kompleksisuusteoria voi valita jonkin tason, koska sen on kuitenkin pakko valita jotain muuta kuin kädet, jotta olisi jokin, mikä tulee olemaan sopiva sen oletetun ongelman ratkaisemiseen.

        Kaikki Theisen reduktion vastaiset lauseet ovat vain kielikukkasia. Hänen oikea tarkoituksensa on odottaa tulevaisuudessa kaikkia hyödyllisiä matemaattisia malleja biologian esittämiseksi.

        Theise sanoi, että sinulla ei ole suolistojörjestelmää, elleivät DNA:t koordinoidu ja järjestäydy bakteerien olemassaoloa varten. Tässä lauseessa DNA on molekyylin kokoinen asia - eikä solun kokoinen.

        Vanha tiede ns. pysähtyi solun kohdalla, koska solua ei voi ns. jakaa tiettyihin asioihin, mitkä näkyvät pikemminkin solua suuremmalla tasolla. Tämä tarkoittaa sitä, että solu rikkoi skaalainvarianssin. Olisi ollut todella väärin jatkaa ajattelua, että esim. lihas kulkee solun läpi samanlaisena lihasmassana. Tällainen tasainen lihas-kontinuumi (löysän mutta kiinteän aineen yhtälöt oletettuna ilman sellaista hilaa, joka olisi mikroskoopin nähtävissä (*)) olisi ollut kuitenkin teoriassa helppo ratkaista, mutta se on kai tässä vain mielikuvituksekkaasti mainittu, eikä sellaisen esim. tulosta ja jotain hyötyjä ole kukaan nähnyt.

        (*) Seuraavassa lauseesa Theise ilmeisesti leikittelee sillä ajatuksella, että jokin solun sisällä oleva asia olisi voinut palvella tämän hilan osana, jolloin kontinuumiteoria tästä hilasta olisi voinut olla jotain, mihin olisi pyritty vähän aikaa. Periaatteessa kukaan ei estä näkemästä myöskään sellaista hilaa, missä on sekoitettu kristalli sekä solun seinää, että solun tumaa, tai kaikkia solun sisäisiä osia erikseen. Mutta syy miksi konseptilla, missä tämä approksimoidaan myös liikeyhtälöksi ei tee mitään, on se, että todella paljon biologisia asioita tapahtuu, kun solut ja niiden osat solun sisällä liikkuvat toistensa suhteen yksilöinä.

        Mainittu Toshio Yanagita käytty mikroskooppia ja katsoi proteiinimolekyylejä (nimenomaan näitä molekyylejä erotettuna koko muusta ruumiista) tarkaan laservalon alla, ja hänestä tuli/tulee lääketieteen mullistaja uuteen kompleksisuuden ja emergenssin aikaan. (Energiamolekyyli ATP ei saa liikettä aikaan kuten joku biologi luuli, vaan se saa aikaan olemassaolevan liikkeen epäsatunnaistumisen. Epäsatunnaistuminen on kuitenkin siitä peräisin edelleen, että ATP myosiini vapauttaa energiaa.)
        https://www.ncbi.nlm.nih.gov/pmc/articles/PMC3521640/
        https://www.researchgate.net/publication/244753006_Model_describing_the_biased_Brownian_movement_of_myosin

        'This article describes how biased Brownian movement and the Brownian search-and-catch mechanism are the basis for muscle contraction. Simulations show that these two mechanisms, as evidenced from single molecule measurements, play a critical role in determining the characteristic properties of muscle....
        Further, it has been demonstrated that myosin motors are typical biological molecular machines that attain their function while under the influence of thermal fluctuations.'

        (Tämä on hyvä esimerkki siitä, miten itseorganisaatiossa on usein kyse jostain, missä entropiaa on paljon.)

        Theisen mukaan 'molekyylit ovat itseorganisoituvia atomeja eivätkä mitään muuta'. Samalla tavalla ihminen on itseorganisoituvia atomeja eikä mitään muuta? (Seuraavaksi hän sanoo, että ihmiskehossa tähän tiettyjen osien organisoitumiseen auttaa DNA, mutta käsitys on varmasti suppea totuus.)

        Itseorganisoituminen on ilmiötason kuvaus mikä ei oikeastaan millään tavalla selitä sitä miksi muodostuu erilaisia systeemisiä rakenteita kuten atomeita, molekyylejä, soluja, eliöitä ja yhteiskuntia, aurinkokuntia, galakseja tai koko maailmankaikeus organisoituneena kokonaisuutena.

        Ei mikään olemassaoleva "luonnonlaki" tai oikeammin fysiikan kaava selitä organisoitumista ja paljon todennäköisempi vaihtoehto olisi nykyisen fysiikkatieteen kannalta ettei minkäänlaisia organisoituneita systeemeitä (eikä meitä kyselemässä) olisi ollenkaan olemassa.

        Luonnon perusvoima kuten sähkömagnetismi ilmenee sekä positiivisena että negatiivisena varauksena. Itse pidän gravitaatiota ja heikkoa ja vahvaa vuorovaikutusta sähkömagnetismin johdannaisian jotka ilmenevät vasta tuon organisoitumisen seurauksena.

        Miten esim. pelikorttien organisoiminen maittain ja numeerisen arvon mukaiseen järjestykseen tai minkä tahansa toimivan tekniikan kehittäminen eroaa tuosta itseorganisoitumisesta ?

        Minusta ei mitenkään koska positiiviset ja negatiiviset sähkövaraukset eivät kykene itse organisoimaan mitään eikä organisoitumaan samaan kokonaisuuteen kuten esim. atomaariseen aineeseen ilman esim. jotain ulkopuolista tai hyperdimensionaalista vaikutusta ja esim. Gabriel Kroninin mukaan kaikki mahdolliset sähkölaitteet edellyttävät hyperdimensionaalisuutta jo pelkästään sen takia että sähkötekniikassa joudutaan käyttämään imaginäärilukuja.

        Kron kehitti systeemien diakoptics metodin monimutkaisten systeemien mallintamiseen
        https://en.wikipedia.org/wiki/Diakoptics

        Reduktionismi on filosofinen käsite jota sovelletaan luonnontieteissä ja kaikki tieteen hypoteesinmuodostus on myös aina läpikotaisin filosofista metafysiikkaa vaikka se olisi matematiikan kielen avulla ilmaistua.

        Fyysikot harrastavat matemaattista taskubiljardia jos yriittävät hahmottaa todellisuutta matematiikka edellä ja filosofit harrastavat hyvin samantapaista käsiteakrobatiaa pelkkien mielikuvien tasolla ja kummallakaan lähestymistavalla ei ole välttämättä ole juuri mitään tekemistä todellisuuden kanssa.

        Kokeellinen tiede ja tekniikan kehitys taas pakottaa käytännön tasolle jossa matematiikka ja kielen käsitteet ovat pelkästään apuvälineitä eikä mikään itsetarkoitus. Toimivuus eli käytännön tason sovellettavuus ovat olennaisin asia eikä teoreettiset loogis-matemaattiset konstruktiot jotka palvelevat ainoastaan kommunikoinnin tasolla.

        Atomien, molekyylien, elinten ja eliöiden tasot vaikuttavat kaikki rinnakkain samanaikaisesti mutta jos eliön keho on terve ja sillä on jonkintasoista kognitiota (kuten jopa yksisoluisilla tohvelieläimillä on todettu) niin sen eliön molekyylit, atomit ja atomien sisäiset entiteetit liikkuvat sinne minne se kognitio kulloinkin ohjaa eikä mikään fysiikan matemaattinen kaava voi kertoi missä ja miten se tapahtuu ja sama koskee vielä suuremmassa määrin monisoluisia kuten ihmisiä jotka eivät ole mitenkään tarkkaanrajattuja kokonaisuuksia edes elinten ja solujen tasolla vaan säilyttävät kokonaisidentieettinsä aktiivisina toimijoina niin kauan kuin heidän kognitionsa on aktiivisessa yhteydessä heidän kehoonsa eli valveilla ollessa.

        Korkeamman organisaatiotason kokonaisuuksia ei kykene kunnolla edes hahmottamaan pelkästään mikroskoopin mittakaavassa eikä mikroskoppi edes kykene tuottamaan relevanttia tietoa ihmisen kokoisesta todellisuudesta kuin vain silloin kun se organisaation ylläpito jostain syystä heikentyy esim. sairauden takia mikä tarkoittaa usein sitä että esim. jokin elin solujensa osalta alkaa ns. "sooloilla" ominpäin.

        Mikä sitten ylläpitää sitä organisaatiotasoa?

        Vaikuttaa siltä että mikään ei oikeasti itseorganisoidu vaan organisointi tulee sen komponentin ulkopuolelta ja todennäköisesti hyperdimensionaaliselta tasolta samaan tapaan kuin Sheldraken ja Beardenin kuvailemissa hahmotuksissa. Kiinteä aine ei kykene itseorgaisoitumaan vaikka nesteet ja kaasut siihen kykenevät joissain olosuhteissa kuten esim. cymatics ilmiössä äänen resonanssien kautta. Samantapainen magneettinen harmooninen resonanssi lienee kaiken mahdollisen itseorganisoitumisen taustalla joka selittänee kaiken itseorganisoitumisen atomeista galaksien tasolle.

        Itseorganisoituminen ilmeisesti mahdollistuu (vaikkei selity) jonkinlaisen hyvin rajoitetun satunnaisuuden avulla minkä takia esim. lähes kaikki esoteriassa tunnetut ennustusmenetelmät (I Ching, Tarot yms.) perustuvat siihen että helposti muokattavasta joukosta ilmenee intention avulla vallitsevat trendit jotka esim. astrologiassa liitetään paikallisen avaruuden konfiguraatioiden ajallis-paikallisiin harmoonisiin resonansseihin.

        ...

        Kiitos asiallisesta kommentistasi!

        B


      • Anonyymi
        Anonyymi kirjoitti:

        Itseorganisoituminen on ilmiötason kuvaus mikä ei oikeastaan millään tavalla selitä sitä miksi muodostuu erilaisia systeemisiä rakenteita kuten atomeita, molekyylejä, soluja, eliöitä ja yhteiskuntia, aurinkokuntia, galakseja tai koko maailmankaikeus organisoituneena kokonaisuutena.

        Ei mikään olemassaoleva "luonnonlaki" tai oikeammin fysiikan kaava selitä organisoitumista ja paljon todennäköisempi vaihtoehto olisi nykyisen fysiikkatieteen kannalta ettei minkäänlaisia organisoituneita systeemeitä (eikä meitä kyselemässä) olisi ollenkaan olemassa.

        Luonnon perusvoima kuten sähkömagnetismi ilmenee sekä positiivisena että negatiivisena varauksena. Itse pidän gravitaatiota ja heikkoa ja vahvaa vuorovaikutusta sähkömagnetismin johdannaisian jotka ilmenevät vasta tuon organisoitumisen seurauksena.

        Miten esim. pelikorttien organisoiminen maittain ja numeerisen arvon mukaiseen järjestykseen tai minkä tahansa toimivan tekniikan kehittäminen eroaa tuosta itseorganisoitumisesta ?

        Minusta ei mitenkään koska positiiviset ja negatiiviset sähkövaraukset eivät kykene itse organisoimaan mitään eikä organisoitumaan samaan kokonaisuuteen kuten esim. atomaariseen aineeseen ilman esim. jotain ulkopuolista tai hyperdimensionaalista vaikutusta ja esim. Gabriel Kroninin mukaan kaikki mahdolliset sähkölaitteet edellyttävät hyperdimensionaalisuutta jo pelkästään sen takia että sähkötekniikassa joudutaan käyttämään imaginäärilukuja.

        Kron kehitti systeemien diakoptics metodin monimutkaisten systeemien mallintamiseen
        https://en.wikipedia.org/wiki/Diakoptics

        Reduktionismi on filosofinen käsite jota sovelletaan luonnontieteissä ja kaikki tieteen hypoteesinmuodostus on myös aina läpikotaisin filosofista metafysiikkaa vaikka se olisi matematiikan kielen avulla ilmaistua.

        Fyysikot harrastavat matemaattista taskubiljardia jos yriittävät hahmottaa todellisuutta matematiikka edellä ja filosofit harrastavat hyvin samantapaista käsiteakrobatiaa pelkkien mielikuvien tasolla ja kummallakaan lähestymistavalla ei ole välttämättä ole juuri mitään tekemistä todellisuuden kanssa.

        Kokeellinen tiede ja tekniikan kehitys taas pakottaa käytännön tasolle jossa matematiikka ja kielen käsitteet ovat pelkästään apuvälineitä eikä mikään itsetarkoitus. Toimivuus eli käytännön tason sovellettavuus ovat olennaisin asia eikä teoreettiset loogis-matemaattiset konstruktiot jotka palvelevat ainoastaan kommunikoinnin tasolla.

        Atomien, molekyylien, elinten ja eliöiden tasot vaikuttavat kaikki rinnakkain samanaikaisesti mutta jos eliön keho on terve ja sillä on jonkintasoista kognitiota (kuten jopa yksisoluisilla tohvelieläimillä on todettu) niin sen eliön molekyylit, atomit ja atomien sisäiset entiteetit liikkuvat sinne minne se kognitio kulloinkin ohjaa eikä mikään fysiikan matemaattinen kaava voi kertoi missä ja miten se tapahtuu ja sama koskee vielä suuremmassa määrin monisoluisia kuten ihmisiä jotka eivät ole mitenkään tarkkaanrajattuja kokonaisuuksia edes elinten ja solujen tasolla vaan säilyttävät kokonaisidentieettinsä aktiivisina toimijoina niin kauan kuin heidän kognitionsa on aktiivisessa yhteydessä heidän kehoonsa eli valveilla ollessa.

        Korkeamman organisaatiotason kokonaisuuksia ei kykene kunnolla edes hahmottamaan pelkästään mikroskoopin mittakaavassa eikä mikroskoppi edes kykene tuottamaan relevanttia tietoa ihmisen kokoisesta todellisuudesta kuin vain silloin kun se organisaation ylläpito jostain syystä heikentyy esim. sairauden takia mikä tarkoittaa usein sitä että esim. jokin elin solujensa osalta alkaa ns. "sooloilla" ominpäin.

        Mikä sitten ylläpitää sitä organisaatiotasoa?

        Vaikuttaa siltä että mikään ei oikeasti itseorganisoidu vaan organisointi tulee sen komponentin ulkopuolelta ja todennäköisesti hyperdimensionaaliselta tasolta samaan tapaan kuin Sheldraken ja Beardenin kuvailemissa hahmotuksissa. Kiinteä aine ei kykene itseorgaisoitumaan vaikka nesteet ja kaasut siihen kykenevät joissain olosuhteissa kuten esim. cymatics ilmiössä äänen resonanssien kautta. Samantapainen magneettinen harmooninen resonanssi lienee kaiken mahdollisen itseorganisoitumisen taustalla joka selittänee kaiken itseorganisoitumisen atomeista galaksien tasolle.

        Itseorganisoituminen ilmeisesti mahdollistuu (vaikkei selity) jonkinlaisen hyvin rajoitetun satunnaisuuden avulla minkä takia esim. lähes kaikki esoteriassa tunnetut ennustusmenetelmät (I Ching, Tarot yms.) perustuvat siihen että helposti muokattavasta joukosta ilmenee intention avulla vallitsevat trendit jotka esim. astrologiassa liitetään paikallisen avaruuden konfiguraatioiden ajallis-paikallisiin harmoonisiin resonansseihin.

        ...

        Kiitos asiallisesta kommentistasi!

        B

        B:
        "Itseorganisoituminen on ilmiötason kuvaus mikä ei oikeastaan millään tavalla selitä sitä miksi muodostuu erilaisia systeemisiä rakenteita kuten atomeita, molekyylejä, soluja, eliöitä ja yhteiskuntia, aurinkokuntia, galakseja tai koko maailmankaikeus organisoituneena kokonaisuutena."

        Ei pidä paikkaansa. Jos on olemassa osia, kuten atomit, eikä tarvitse vastata kysymykseen, miksi atomeja on, niin sen jälkeen kaikki, mitä atomeista tulee, on miksivastattu jollakin itseorganisaation kaltaisella atomien evoluution kuvauksella. Myös on vastattu sen suhteen, että niillä on rakenne. Koska rakenne on vain tietty atomien olotila. Jotkut itseorganisaatiomallit olisivat niin hyviä, että ne ennustaisivat rakenteen tulevan aina olemaan, jos aloitettaisiin atomeista ilman rakennetta. Eli ne ennustavat, että rakenne on ainoa mahdolisuus, jos atomeja on kuviteltu olevan.

        B:
        "Ei mikään olemassaoleva "luonnonlaki" tai oikeammin fysiikan kaava selitä organisoitumista..."

        Sanoit äsken ettei (itse)organisoituminen ole sinusta 'mitään'. Miksi pitäisi ensinnäkään pyrkiä selittämään asia, joka ei ole mitään?

        B:
        "Luonnon perusvoima kuten sähkömagnetismi ilmenee sekä positiivisena että negatiivisena varauksena. Itse pidän gravitaatiota ja heikkoa ja vahvaa vuorovaikutusta sähkömagnetismin johdannaisian jotka ilmenevät vasta tuon organisoitumisen seurauksena."

        Jos tarkoitat, että aine on organisoitunut tai ei ole, ja että on olemassa vain sähkömagneettinen aine, eikä mitään muuta vuorovaikutusta, niin ei kannata esittää, että sen pitää olla organisoitunut ennen kuin mitään ilmenee gravitaatiofyysikoille. On olemassa todennäköisesti miljoona tapaa ja tapausta tälläkin hetkellä näkyvissä, missä SM-aine joutuisi epäorganisoitumaan ja silloin todellisuus rakoilisi siellä sun täällä. Keksi ensin, miten tehdään gravitaatioefekti sähköllä ja sano sitten vasta, mikä estää tekemästä gravitaatioefektejä epäorganisoituneesta SM-aineesta.

        Keskustelu siitä, onko SM-aine organisoituneessa muodossaan gravitaatioefekti, ei muuten mitenkään anna olettaa, etteikö jonkun täysin toisen ihmisen gravitaatioteoria ja aine, missä on läsnä vain tämä gravitaatio, voisi muodostaa organisoituvaa ainetta. Jossain sanan merkityksessä, missä se ei tarkoita molekyylejä.

        Vai tarkoitatko sitä, että koska varauksia ei ole kolmea tai yhtä kappaletta satunnaisesti, niin varaukset ovat organisoineet jotain? Fysiikan lait on tehty varauksille sillä tavalla, että ne eivät koskaan muutu ajassa (ei lasketa mitään faasitransition läpi muuttuvaa asiaa muutokseksi, eikä mitään yhtenäiskenttäteoriaa, missä varausten määrät kerrotaan). Tämän takia kukaan ei ole yrittänyt itseorganisoida niitä. Jos joku esittää joskus ideanaan sen, että varausten lukumäärä vaihtelee yhden elektronin luona esim. koko ajan (tai jotain ihan muuta, missä ei kiinnitetä sen elektronin olemassaoloa), niin silloin on itseorganisoijillekin tehtävää, ja he saattavat alkaa etsiä tästä esitetystä ideasta erilaisten sen mukaan toimivien usean kappaleen systeemien käytöstä, mistä sitten voi katsoa organisoituuko joku joskus juuri kahdeksi varaukseksi. Kysymys on sitten siitä, että voiko ideana annettu sääntö useammista-varauksista-yhtäaikaa sisältää kaiken tiedon siitä, miksi sellaiset asiat itseorganisoituvat. Arvaa miten väärässä olet, jos näin kävisi. Ja jos ei käy, niin voiko joku kuitenkin parannella sitä ideaa sellaiseen suuntaan.

        B:
        "Miten esim. pelikorttien organisoiminen maittain ja numeerisen arvon mukaiseen järjestykseen tai minkä tahansa toimivan tekniikan kehittäminen eroaa tuosta itseorganisoitumisesta ?"

        Monissa molekyylien itseorganisaatioissa kyse on siitä, että pelikortit eivät ole neutraaleja toisilleen, vaan jotkut niistä tarttuvat yhteen. Sitä ennen pelikortit pitää heittää johonkin nesteeseen, missä ne kelluvat, mutta missä ne eivät saa pysyä paikallaan. Biologia voisi olla mahdotonta, jos esim. vesimolekyylin ja proteiinimolekyylin (tai sen aineen, mistä aletaan rakentaa proteiinia) välinen kokoero olisi yli miljardiluokkaa kertaa nykyinen ero, ja proteiini kelluisi kortin lailla paikoillaan ja reagoisi vain makroskooppisiin aaltoihin ja virtaan.

        B:
        "Minusta ei mitenkään koska positiiviset ja negatiiviset sähkövaraukset eivät kykene itse organisoimaan mitään..."

        Siksi on olemassa kvanttimekaniikka.

        Lauseesi on todiste sähkövaraustodellisuutta vastaan. Ennen kuin mitään organisoidaan, täytyy olla käsitys liikelaista - mitä ne asiat, missä on varaus, noudattavat. Siinä on liikelaki usealle kappaleelle ja vuorovaikutuksen vaikutustapa kaikille eri mahdollisuuksille, missä varaukset ovat toisiinsa nähden. Sinun tulisi arvioida jokainen sellainen läpi erikseen.

        1


      • Anonyymi
        Anonyymi kirjoitti:

        B:
        "Itseorganisoituminen on ilmiötason kuvaus mikä ei oikeastaan millään tavalla selitä sitä miksi muodostuu erilaisia systeemisiä rakenteita kuten atomeita, molekyylejä, soluja, eliöitä ja yhteiskuntia, aurinkokuntia, galakseja tai koko maailmankaikeus organisoituneena kokonaisuutena."

        Ei pidä paikkaansa. Jos on olemassa osia, kuten atomit, eikä tarvitse vastata kysymykseen, miksi atomeja on, niin sen jälkeen kaikki, mitä atomeista tulee, on miksivastattu jollakin itseorganisaation kaltaisella atomien evoluution kuvauksella. Myös on vastattu sen suhteen, että niillä on rakenne. Koska rakenne on vain tietty atomien olotila. Jotkut itseorganisaatiomallit olisivat niin hyviä, että ne ennustaisivat rakenteen tulevan aina olemaan, jos aloitettaisiin atomeista ilman rakennetta. Eli ne ennustavat, että rakenne on ainoa mahdolisuus, jos atomeja on kuviteltu olevan.

        B:
        "Ei mikään olemassaoleva "luonnonlaki" tai oikeammin fysiikan kaava selitä organisoitumista..."

        Sanoit äsken ettei (itse)organisoituminen ole sinusta 'mitään'. Miksi pitäisi ensinnäkään pyrkiä selittämään asia, joka ei ole mitään?

        B:
        "Luonnon perusvoima kuten sähkömagnetismi ilmenee sekä positiivisena että negatiivisena varauksena. Itse pidän gravitaatiota ja heikkoa ja vahvaa vuorovaikutusta sähkömagnetismin johdannaisian jotka ilmenevät vasta tuon organisoitumisen seurauksena."

        Jos tarkoitat, että aine on organisoitunut tai ei ole, ja että on olemassa vain sähkömagneettinen aine, eikä mitään muuta vuorovaikutusta, niin ei kannata esittää, että sen pitää olla organisoitunut ennen kuin mitään ilmenee gravitaatiofyysikoille. On olemassa todennäköisesti miljoona tapaa ja tapausta tälläkin hetkellä näkyvissä, missä SM-aine joutuisi epäorganisoitumaan ja silloin todellisuus rakoilisi siellä sun täällä. Keksi ensin, miten tehdään gravitaatioefekti sähköllä ja sano sitten vasta, mikä estää tekemästä gravitaatioefektejä epäorganisoituneesta SM-aineesta.

        Keskustelu siitä, onko SM-aine organisoituneessa muodossaan gravitaatioefekti, ei muuten mitenkään anna olettaa, etteikö jonkun täysin toisen ihmisen gravitaatioteoria ja aine, missä on läsnä vain tämä gravitaatio, voisi muodostaa organisoituvaa ainetta. Jossain sanan merkityksessä, missä se ei tarkoita molekyylejä.

        Vai tarkoitatko sitä, että koska varauksia ei ole kolmea tai yhtä kappaletta satunnaisesti, niin varaukset ovat organisoineet jotain? Fysiikan lait on tehty varauksille sillä tavalla, että ne eivät koskaan muutu ajassa (ei lasketa mitään faasitransition läpi muuttuvaa asiaa muutokseksi, eikä mitään yhtenäiskenttäteoriaa, missä varausten määrät kerrotaan). Tämän takia kukaan ei ole yrittänyt itseorganisoida niitä. Jos joku esittää joskus ideanaan sen, että varausten lukumäärä vaihtelee yhden elektronin luona esim. koko ajan (tai jotain ihan muuta, missä ei kiinnitetä sen elektronin olemassaoloa), niin silloin on itseorganisoijillekin tehtävää, ja he saattavat alkaa etsiä tästä esitetystä ideasta erilaisten sen mukaan toimivien usean kappaleen systeemien käytöstä, mistä sitten voi katsoa organisoituuko joku joskus juuri kahdeksi varaukseksi. Kysymys on sitten siitä, että voiko ideana annettu sääntö useammista-varauksista-yhtäaikaa sisältää kaiken tiedon siitä, miksi sellaiset asiat itseorganisoituvat. Arvaa miten väärässä olet, jos näin kävisi. Ja jos ei käy, niin voiko joku kuitenkin parannella sitä ideaa sellaiseen suuntaan.

        B:
        "Miten esim. pelikorttien organisoiminen maittain ja numeerisen arvon mukaiseen järjestykseen tai minkä tahansa toimivan tekniikan kehittäminen eroaa tuosta itseorganisoitumisesta ?"

        Monissa molekyylien itseorganisaatioissa kyse on siitä, että pelikortit eivät ole neutraaleja toisilleen, vaan jotkut niistä tarttuvat yhteen. Sitä ennen pelikortit pitää heittää johonkin nesteeseen, missä ne kelluvat, mutta missä ne eivät saa pysyä paikallaan. Biologia voisi olla mahdotonta, jos esim. vesimolekyylin ja proteiinimolekyylin (tai sen aineen, mistä aletaan rakentaa proteiinia) välinen kokoero olisi yli miljardiluokkaa kertaa nykyinen ero, ja proteiini kelluisi kortin lailla paikoillaan ja reagoisi vain makroskooppisiin aaltoihin ja virtaan.

        B:
        "Minusta ei mitenkään koska positiiviset ja negatiiviset sähkövaraukset eivät kykene itse organisoimaan mitään..."

        Siksi on olemassa kvanttimekaniikka.

        Lauseesi on todiste sähkövaraustodellisuutta vastaan. Ennen kuin mitään organisoidaan, täytyy olla käsitys liikelaista - mitä ne asiat, missä on varaus, noudattavat. Siinä on liikelaki usealle kappaleelle ja vuorovaikutuksen vaikutustapa kaikille eri mahdollisuuksille, missä varaukset ovat toisiinsa nähden. Sinun tulisi arvioida jokainen sellainen läpi erikseen.

        1

        B:
        "...eikä organisoitumaan samaan kokonaisuuteen kuten esim. atomaariseen aineeseen ilman esim. jotain ulkopuolista tai hyperdimensionaalista vaikutusta ja esim. Gabriel Kroninin mukaan kaikki mahdolliset sähkölaitteet edellyttävät hyperdimensionaalisuutta jo pelkästään sen takia että sähkötekniikassa joudutaan käyttämään imaginäärilukuja."

        Varaukset eli ne varaukset, jotka liikkuvat, ovat jo imaginäärilukuja. Tarkemmin sanottuna neljä sellaista. Jos varauksissa käytetään jo imaginäärilukuja, ne ovat dimensionaalisuus X. Jos ne eivät ole itseorganisoituvia koskaan, silloin X-dimensionalisuus ei auta niitä organisoitumaan, kun dimensionalisuutta X vaatii sille. Tämä minun imaginäärilukuvaraus tulee siitä, että periaatteessa käytetään kvanttimekaniikkaa eikä Newtonin mekaniikkaa. Varauksella itsellään ei ole paljon annettavaa, kun nämä eri mekaniikat voivat viedä sitä, joten riippumatta siitä, mitä mekaniikassa on, voisi olla mahdollisuus tehdä jotain organisoituvaa (esim. pelkän gravitaation aine yllä).

        Jos käytät QM:n neljän imaginääriluvun varausta tekemään itseorganisaation selittäviä aineen malleja, niin olet korkein reduktionisti (eli pienimmän koon) tähän mennessä.

        Kompleksilukujen organisaatio akseleittain ei eroa mitenkään korttipakan korttien organisoitumisesta.

        Sähkövaraukset rakentavat asioita, kuten DNA, mutta miksi aletaan puhua virtapiireistä? Jos puhutaan sähkövarauksista virtapiirissä, jonka teki ihminen (tai electricuniversen luojajumala), ei ole mitään itseorganisaatiota. Kyseessä on paljon pakko-organisaatiota, koska joku on tehnyt kuparimalmista kuparilankoja, jotka ovat puhdistettua metallia, ja joissa on metrien pituisia matkoja, joissa aineen ominaisuudet sähkönjohtamiselle ovat kaikkialla tasaiset. Tämä auttaa pitämään myös kaikki virtaan osallistuvat elektronit tasaisessa liikkeessä eli ikäänkuin parvessa. Kaikki mikä ei pysy parvessa, on liian merkityksetöntä vaikuttaakseen minkäälaiseen laitteen toimintaan, ja tarvitsisi mikroskooppimaista tarkkuuta näkyäkseen kenellekään. Ajatteletko, että jos joku antaa kuparilangat sinulle, ja jos kytket ne paristoon, silloin tarvitaan vielä jotain muuta, koska muutoin esim. elektronit menevät aineessa edelleen sekaisin vääriin suuntiin niinkuin kaasussa, joka ammutaan tyhjiöön tai toiseen kaasuun, ja se leviää heti pariston navan luona ja vielä sekaisemmin kauemmas joutuessaan? Käytätkö jotain kompleksista asiaa materiaalin ja sen synnyn kuvaamiseen, vai enemmän liikkumassa olevan elektronin kuvaamiseen?

        https://en.wiktionary.org/wiki/hyperdimensional
        Sanakirjamääritelmä hyperdimensionaaliselle voisi olla mielummin, että avaruusdimensioita on _eri verran_ kuin miltä universumi näytti 1900-luvun pääsuuntien fysiikassa. Silloin olisi mahdollista sanoa, että 1-kertainen kompleksiavaruus, missä on reaalisuunta ja imaginäärisuunta, on hyperdimensionaalinen. Kompleksiavaruudessa eläminen on aivan sama kuin 2D-avaruudessa eläminen, koska voidaan ottaa kompleksiavaruuden pisteiden etäisyys tunnetun etäisyyden määritelmän tilalle. Tämän jälkeen elämässä ei ole mitään avaruuden laskutoimituksia, missä avaruuden piste kerrotaan toisella avaruuden pisteellä, ja missä sen laskun tulos on elämän määritelmä.

        Matematiikka on laji, missä esim. asia kuten reaalisuora voi esiintyä monessa eri yhteydessä ja monen eri dimensioisen kappaleen sisällä. Mitä tahansa näistä voidaan käyttää hyväksi, jos halutaan tietää jotain yhdeltä reaalisuoralta. Sen takia on joskus olemassa ongelmia, missä dimensioita lisätään apudimensioiksi, koska apudimensioissa on helppo laskea (kompleksiluvuissa laskeminen jopa on erilaista kuin reaaliluvuissa). Tästä erikseen on myös jotkut ongelmat, missä ihmisen ensimmäinen johtopäätös on, että dimensioita on yksi, mutta missä ongelman esitys ja ratkaisu on jotain, mikä olisi tietynlainen kahden dimension ongelma, eikä toista niistä jätetä lopussa edes pois siten, että informaatio sieltä katoaisi kuten äsken. Tällainen on esim. ajan liittäminen aika-avaruuteen tai nopeuden liittäminen faasiavaruuteen.

        Virtapiirilaskuissa sinut on saatu sekoittamaan keskenään kaksi henkilöä. Vuonna 1893 Arthur Kennelly käytti ensimmäistä kertaa kompleksilukuja esittämään virtapiirin impedanssia. Tämä impedanssi voi korvata kaikki resistanssit ja kapasitanssit, ja sillä on omat laskukeinonsa, jotka muistuttavat esim. Ohmin yksinkertaistettua laskukeinoa. Koska kaikki oletuksia käyttävät virtapiirit voidaan muuttaa impedansseiksi, niin nämä kompleksiluvut ovat sitä jotain, mitä voi sanoa käytettävän yleisesti missä tahansa yksinkertaisessa virtapiirissä. Kronin työ ei koske mitään helppoa virtapiiriä, missä impedansseja on vähän. Kun Kenelly laski kompleksiluvuissa, hän ei tehnyt sitä siksi, että dimensio piti saada samalla tavalla kuin mitä äskeisessä kappaleessa haetaan takaa, vaan siksi, että perus- kahden objektin välinen algebrallinen laskutoimitus siniaalloille oli helpompi (seuraavalla videolla).

        2


      • Anonyymi
        Anonyymi kirjoitti:

        B:
        "...eikä organisoitumaan samaan kokonaisuuteen kuten esim. atomaariseen aineeseen ilman esim. jotain ulkopuolista tai hyperdimensionaalista vaikutusta ja esim. Gabriel Kroninin mukaan kaikki mahdolliset sähkölaitteet edellyttävät hyperdimensionaalisuutta jo pelkästään sen takia että sähkötekniikassa joudutaan käyttämään imaginäärilukuja."

        Varaukset eli ne varaukset, jotka liikkuvat, ovat jo imaginäärilukuja. Tarkemmin sanottuna neljä sellaista. Jos varauksissa käytetään jo imaginäärilukuja, ne ovat dimensionaalisuus X. Jos ne eivät ole itseorganisoituvia koskaan, silloin X-dimensionalisuus ei auta niitä organisoitumaan, kun dimensionalisuutta X vaatii sille. Tämä minun imaginäärilukuvaraus tulee siitä, että periaatteessa käytetään kvanttimekaniikkaa eikä Newtonin mekaniikkaa. Varauksella itsellään ei ole paljon annettavaa, kun nämä eri mekaniikat voivat viedä sitä, joten riippumatta siitä, mitä mekaniikassa on, voisi olla mahdollisuus tehdä jotain organisoituvaa (esim. pelkän gravitaation aine yllä).

        Jos käytät QM:n neljän imaginääriluvun varausta tekemään itseorganisaation selittäviä aineen malleja, niin olet korkein reduktionisti (eli pienimmän koon) tähän mennessä.

        Kompleksilukujen organisaatio akseleittain ei eroa mitenkään korttipakan korttien organisoitumisesta.

        Sähkövaraukset rakentavat asioita, kuten DNA, mutta miksi aletaan puhua virtapiireistä? Jos puhutaan sähkövarauksista virtapiirissä, jonka teki ihminen (tai electricuniversen luojajumala), ei ole mitään itseorganisaatiota. Kyseessä on paljon pakko-organisaatiota, koska joku on tehnyt kuparimalmista kuparilankoja, jotka ovat puhdistettua metallia, ja joissa on metrien pituisia matkoja, joissa aineen ominaisuudet sähkönjohtamiselle ovat kaikkialla tasaiset. Tämä auttaa pitämään myös kaikki virtaan osallistuvat elektronit tasaisessa liikkeessä eli ikäänkuin parvessa. Kaikki mikä ei pysy parvessa, on liian merkityksetöntä vaikuttaakseen minkäälaiseen laitteen toimintaan, ja tarvitsisi mikroskooppimaista tarkkuuta näkyäkseen kenellekään. Ajatteletko, että jos joku antaa kuparilangat sinulle, ja jos kytket ne paristoon, silloin tarvitaan vielä jotain muuta, koska muutoin esim. elektronit menevät aineessa edelleen sekaisin vääriin suuntiin niinkuin kaasussa, joka ammutaan tyhjiöön tai toiseen kaasuun, ja se leviää heti pariston navan luona ja vielä sekaisemmin kauemmas joutuessaan? Käytätkö jotain kompleksista asiaa materiaalin ja sen synnyn kuvaamiseen, vai enemmän liikkumassa olevan elektronin kuvaamiseen?

        https://en.wiktionary.org/wiki/hyperdimensional
        Sanakirjamääritelmä hyperdimensionaaliselle voisi olla mielummin, että avaruusdimensioita on _eri verran_ kuin miltä universumi näytti 1900-luvun pääsuuntien fysiikassa. Silloin olisi mahdollista sanoa, että 1-kertainen kompleksiavaruus, missä on reaalisuunta ja imaginäärisuunta, on hyperdimensionaalinen. Kompleksiavaruudessa eläminen on aivan sama kuin 2D-avaruudessa eläminen, koska voidaan ottaa kompleksiavaruuden pisteiden etäisyys tunnetun etäisyyden määritelmän tilalle. Tämän jälkeen elämässä ei ole mitään avaruuden laskutoimituksia, missä avaruuden piste kerrotaan toisella avaruuden pisteellä, ja missä sen laskun tulos on elämän määritelmä.

        Matematiikka on laji, missä esim. asia kuten reaalisuora voi esiintyä monessa eri yhteydessä ja monen eri dimensioisen kappaleen sisällä. Mitä tahansa näistä voidaan käyttää hyväksi, jos halutaan tietää jotain yhdeltä reaalisuoralta. Sen takia on joskus olemassa ongelmia, missä dimensioita lisätään apudimensioiksi, koska apudimensioissa on helppo laskea (kompleksiluvuissa laskeminen jopa on erilaista kuin reaaliluvuissa). Tästä erikseen on myös jotkut ongelmat, missä ihmisen ensimmäinen johtopäätös on, että dimensioita on yksi, mutta missä ongelman esitys ja ratkaisu on jotain, mikä olisi tietynlainen kahden dimension ongelma, eikä toista niistä jätetä lopussa edes pois siten, että informaatio sieltä katoaisi kuten äsken. Tällainen on esim. ajan liittäminen aika-avaruuteen tai nopeuden liittäminen faasiavaruuteen.

        Virtapiirilaskuissa sinut on saatu sekoittamaan keskenään kaksi henkilöä. Vuonna 1893 Arthur Kennelly käytti ensimmäistä kertaa kompleksilukuja esittämään virtapiirin impedanssia. Tämä impedanssi voi korvata kaikki resistanssit ja kapasitanssit, ja sillä on omat laskukeinonsa, jotka muistuttavat esim. Ohmin yksinkertaistettua laskukeinoa. Koska kaikki oletuksia käyttävät virtapiirit voidaan muuttaa impedansseiksi, niin nämä kompleksiluvut ovat sitä jotain, mitä voi sanoa käytettävän yleisesti missä tahansa yksinkertaisessa virtapiirissä. Kronin työ ei koske mitään helppoa virtapiiriä, missä impedansseja on vähän. Kun Kenelly laski kompleksiluvuissa, hän ei tehnyt sitä siksi, että dimensio piti saada samalla tavalla kuin mitä äskeisessä kappaleessa haetaan takaa, vaan siksi, että perus- kahden objektin välinen algebrallinen laskutoimitus siniaalloille oli helpompi (seuraavalla videolla).

        2

        Näytetään seuraavaksi että kompleksilukujen valinta juuri Kennelyn tavalla on tiedon kannalta epäpakollinen, mielivaltainen ja täysin neutraali toimenpide, joka ei vaikuta todellisuuden tulkitsemiseen. Viestissäsi lukee jotain myös siitä, että toimivuus on hyvä asia verrattuna matematiikkaan. Näiden piirien matematiikka käydän tässä nyt läpi ja näytetään, ettei niissä toimivuudesta seuraa sitä, että oltaisiin todellisia ja tietysti ettei kyseessä ole mikään erikoinen emergenssi, joka olisi viite ei-reduktionismiin.

        Impedanssi ja kompleksiluku RC-piirissä yksinkertaistaa laskua verrattuna kapasitanssin käyttöön:
        https://www.youtube.com/watch?v=gafnlatW8jw

        Kannattaa saada itselleen rehellinen kuva siitä, millaisia oletuksia on tehnyt taustalla, kun esittää jotain virtapiiristä. Tässä videossa oli sanottu joitain niistä. Sen mukaan alussa on lineaariset differentiaaliyhtälöt kapasitanssin kanssa. Kyseessä ei ole vielä todellisuus, vaan tämäkin perustuu enimmälti oletukseen siitä, että sähkömagneettinen objekti on virrantiheys, jota kulkee aina sähkökentän suuntaan, ja että tätä virtaa tarvitsee tutkia vain yksittäisissä kohdissa komponenttien luona sillä muualla virta on samaa. Näistä syvemmistä oletuksista lisää seuraavissa kappaleissa. Videolla seuraavat esitettävät oletukset ovat, että differentiaaliyhtälöiden mukaisesti kehittyvässä piirissä syntyy tasapainotilanne, kun t menee äärettömään. Vaihtovirtapiirissäkin vain tätä lopputilannetta tarkastellaan eikä impedanssin käyttämisestä ja kompleksilukujen käyttämisestä ole hyötyä, jos haluaa tietää, mitä tapahtuu ensimmäisen nanosekunnin aikana. Malli joka sellaisena kompleksisena impedanssina esitetään, on siis tuollaisessa muodossaan tietämätön siitä, mitä tapahtuu kaikkina hetkinä. Jos ajattelisi vertailun vuoksi olevan olemassa jokin piiri missä kulkee nestettä, niin tässä on käytetty myös hyväksi sitä, että neste kulkee nopeasti tai sähkökentät asettuvat nopeasti verrattuna siihen, mikä on tarvittu käyttöönottoaika.

        Kron on ilmeisesti keskittynyt johonkin muuhun monimutkaisuuteen kuin kaikkien hetkien tietämiseen differentiaaliyhtälöiden avulla.

        Yksittäinen differentiaaliyhtälö, jossa ei mainita paikkaa avaruudessa, kuvaa koko tällaista piiriä, koska ongelmassa oletetaan, että riittää tietää, miten varaus kerääntyy sisään tulevan muuttuvan virran muodossa kondensaattorin toiseen esim. levyyn. Sitten yhtälön termi tulee siitä, mikä on sähkökentän muoto ja potentiaaliero näiden levyjen välissä. Tästä tulee oletuksen mukaan suoraan tieto, mitä tavallisessa johdossa ympärillä pitää tapahtua, koska potentiaaliero, joka sitten on tasainen kaikkialla liikuttaa siellä vain varauksia. Kun mitään muuta kuin komponentin pieni alue ei tarvitse tietää ja sitä pidetään eristettynä muusta maailmasta ja maailma eristetään myös siltä, puhutaan sellaisesta approksimaatiosta kuin Lumped Circuit Analysis, joka äskeisessä videossa on jo oletettu aiemmin.

        Lumped-element model:
        https://www.youtube.com/watch?v=d-a9Pr2z-qg
        -Viidenteen minuuttiin mennessä perusoletuksen merkitys sanottuna hyvin, ja myös miten se ei pidä paikkaansa

        MIT:
        https://www.youtube.com/watch?v=AfQxyVuLeCs&list=PL9F74AFA03AA06A11&index=1
        -Videolla tehdään kahdesti (21:12 ja 38:22) Maxwellin yhtälöistä johdettu näiden yhtälöiden versio, missä helpompi sääntö pätee. Ensimmäinen näistä on Ohmin laki. Alussa ei olisi kannattanut olla Ohmin laki samassa 'yksinkertaisuuden' laatikossa kuin Maxwell.
        -35:00 on myös esimerkkejä siitä ettei yksinkertaistus pidä aina paikkaansa.
        -Sarjan 2. osan alku on vielä selväsanaisempi viesti sille, mikä on Lumped Circuit Analysisin virallinen muoto.

        Näissä Maxwellin yhtälöissä on sellainen versio, missä oletetaan edelleen, että sähkökentän lähde on voitu kirjoittaa virrantiheydeksi. Yleinen virrantiheys olisi paikasta riippuva vektorikenttä. Miksi tiheytenä jo annettu elektronikaasu liikkuu juuri kuten virta sähkökentän suuntaan, on mallinnettu näin:
        https://en.wikipedia.org/wiki/Drude_model
        https://en.wikipedia.org/wiki/Free_electron_model
        Näissä kaasuissa on tyypillisiä kaasuoletuksia, mikä ei täysin vastaa elektronien todellisuutta, missä vain yksittäiset hiukkaset ovat Maxwellin sähkö-kentän lähteitä.

        3


      • Anonyymi
        Anonyymi kirjoitti:

        Näytetään seuraavaksi että kompleksilukujen valinta juuri Kennelyn tavalla on tiedon kannalta epäpakollinen, mielivaltainen ja täysin neutraali toimenpide, joka ei vaikuta todellisuuden tulkitsemiseen. Viestissäsi lukee jotain myös siitä, että toimivuus on hyvä asia verrattuna matematiikkaan. Näiden piirien matematiikka käydän tässä nyt läpi ja näytetään, ettei niissä toimivuudesta seuraa sitä, että oltaisiin todellisia ja tietysti ettei kyseessä ole mikään erikoinen emergenssi, joka olisi viite ei-reduktionismiin.

        Impedanssi ja kompleksiluku RC-piirissä yksinkertaistaa laskua verrattuna kapasitanssin käyttöön:
        https://www.youtube.com/watch?v=gafnlatW8jw

        Kannattaa saada itselleen rehellinen kuva siitä, millaisia oletuksia on tehnyt taustalla, kun esittää jotain virtapiiristä. Tässä videossa oli sanottu joitain niistä. Sen mukaan alussa on lineaariset differentiaaliyhtälöt kapasitanssin kanssa. Kyseessä ei ole vielä todellisuus, vaan tämäkin perustuu enimmälti oletukseen siitä, että sähkömagneettinen objekti on virrantiheys, jota kulkee aina sähkökentän suuntaan, ja että tätä virtaa tarvitsee tutkia vain yksittäisissä kohdissa komponenttien luona sillä muualla virta on samaa. Näistä syvemmistä oletuksista lisää seuraavissa kappaleissa. Videolla seuraavat esitettävät oletukset ovat, että differentiaaliyhtälöiden mukaisesti kehittyvässä piirissä syntyy tasapainotilanne, kun t menee äärettömään. Vaihtovirtapiirissäkin vain tätä lopputilannetta tarkastellaan eikä impedanssin käyttämisestä ja kompleksilukujen käyttämisestä ole hyötyä, jos haluaa tietää, mitä tapahtuu ensimmäisen nanosekunnin aikana. Malli joka sellaisena kompleksisena impedanssina esitetään, on siis tuollaisessa muodossaan tietämätön siitä, mitä tapahtuu kaikkina hetkinä. Jos ajattelisi vertailun vuoksi olevan olemassa jokin piiri missä kulkee nestettä, niin tässä on käytetty myös hyväksi sitä, että neste kulkee nopeasti tai sähkökentät asettuvat nopeasti verrattuna siihen, mikä on tarvittu käyttöönottoaika.

        Kron on ilmeisesti keskittynyt johonkin muuhun monimutkaisuuteen kuin kaikkien hetkien tietämiseen differentiaaliyhtälöiden avulla.

        Yksittäinen differentiaaliyhtälö, jossa ei mainita paikkaa avaruudessa, kuvaa koko tällaista piiriä, koska ongelmassa oletetaan, että riittää tietää, miten varaus kerääntyy sisään tulevan muuttuvan virran muodossa kondensaattorin toiseen esim. levyyn. Sitten yhtälön termi tulee siitä, mikä on sähkökentän muoto ja potentiaaliero näiden levyjen välissä. Tästä tulee oletuksen mukaan suoraan tieto, mitä tavallisessa johdossa ympärillä pitää tapahtua, koska potentiaaliero, joka sitten on tasainen kaikkialla liikuttaa siellä vain varauksia. Kun mitään muuta kuin komponentin pieni alue ei tarvitse tietää ja sitä pidetään eristettynä muusta maailmasta ja maailma eristetään myös siltä, puhutaan sellaisesta approksimaatiosta kuin Lumped Circuit Analysis, joka äskeisessä videossa on jo oletettu aiemmin.

        Lumped-element model:
        https://www.youtube.com/watch?v=d-a9Pr2z-qg
        -Viidenteen minuuttiin mennessä perusoletuksen merkitys sanottuna hyvin, ja myös miten se ei pidä paikkaansa

        MIT:
        https://www.youtube.com/watch?v=AfQxyVuLeCs&list=PL9F74AFA03AA06A11&index=1
        -Videolla tehdään kahdesti (21:12 ja 38:22) Maxwellin yhtälöistä johdettu näiden yhtälöiden versio, missä helpompi sääntö pätee. Ensimmäinen näistä on Ohmin laki. Alussa ei olisi kannattanut olla Ohmin laki samassa 'yksinkertaisuuden' laatikossa kuin Maxwell.
        -35:00 on myös esimerkkejä siitä ettei yksinkertaistus pidä aina paikkaansa.
        -Sarjan 2. osan alku on vielä selväsanaisempi viesti sille, mikä on Lumped Circuit Analysisin virallinen muoto.

        Näissä Maxwellin yhtälöissä on sellainen versio, missä oletetaan edelleen, että sähkökentän lähde on voitu kirjoittaa virrantiheydeksi. Yleinen virrantiheys olisi paikasta riippuva vektorikenttä. Miksi tiheytenä jo annettu elektronikaasu liikkuu juuri kuten virta sähkökentän suuntaan, on mallinnettu näin:
        https://en.wikipedia.org/wiki/Drude_model
        https://en.wikipedia.org/wiki/Free_electron_model
        Näissä kaasuissa on tyypillisiä kaasuoletuksia, mikä ei täysin vastaa elektronien todellisuutta, missä vain yksittäiset hiukkaset ovat Maxwellin sähkö-kentän lähteitä.

        3

        Kun tehdään jokin MIT:in mainitsema suurempi komponentti, se on mahdollista siksi, että on löydetty Maxwellin yhtälöiden mukainen tapaus, missä todellisuus komponentissa on Maxwellin yhtälöiden tilana todella erikoisessa järjetyksessä, joka on pysyvä. Komponentin tuottaminen ei ole aina mahdollista, jos ei yhtään tutki asiaa Maxwellin yhtälöiden tasolla, koska vain Maxwellin yhtälöiden tasolla voi nähdä, onko se täyttynyt häiriöistä ja esim. termeistä, jotka eivät ole toivottuja nollia. Vain etsimällä ja mittaamalla sellaisia termejä voi tietää, että ne ovat nollia ja että nykyinen rakennelma tulee rakentumaan hyödylliseksi. Tekstissäsi mainitut suuret käytännön hyödyt tulevat yleensä siitä, että on tuhansia tai miljoonia piirin komponentteja. Vain yhdessä niillä on jokin kodinkoneen merkitys. Mutta se miten tähän hyödyllisyyteen on päästy, on joskus ollut se, että nämä komponentit on pystytty yksitellen tuottamaan ja antamaan kokoonpanon suunnittelijan käyttöön muodossa, joka on ideaaliseksi todettu Maxwellin tilana. Toisaalta esim. ideaalisen kondensaattorin pystyy myös tekemään tuurilla ja intuitiolla, koska siinä ideaalisuus toteutuu ottamalla homogeenista puhdasta ainetta ja tekemällä siitä geometrisesti sileän muotoisen ja suoran. Tälläinen aineen yksinkertaistus 3D-avaruudessa johtaa kyseiseen yksinkertaiseen Maxwell-tilaan, jota sitten saa käyttää helpompien sääntöjen mukaan.

        Kun yllä kysytiin, että oletatko elektronien leviävän kaasuna virtapiiriin, niin tämä on jollain tavalla oletus, joka on ristiriidassa sen kanssa, mitä virtapiirilait olettavat olevan mahdollista siinä aineessa, mitä ne olettavat virtapiirin olevan. Eli sinun oletuksesi mukaan nämä lait pitäisi poistaa, ja uudet lait tehtäisiin sen mukaan mitä materiaalissa esim. kuvittelee tai näkee olevan, minkä jälkeen siitä kirjoitetaan jonkinlainen Maxwellin yhtälöiden mukainen elektronien ja niiden ympäristön välinen liikkumisen muoto.

        Kron ei ole sanonut koskaan, että avaruus on kompleksiavaruus. Sen sijaan hän on käyttänyt 3D-avaruudessa (tai kokonaan avaruuden ulkopuolella, koska usein laskuissa ei ole tarkoitus edetä avaruudessa) sellaisia objekteja, jotka ovat kompleksisia. Objektit, joita hän käytti eivät ole aina kompleksisia, joten ei ole totta, että Kron edellyttää kaikkien hänen sähkökohteidensa olevan ongelmassa kompleksilukujen muodossa. Lisäksi Kron on tehnyt ongelmista varmaan monen dimension objekteja, joiden avaruus on kuitenkin tavallinen. Tai sitten hän on lisäksi tehnyt jotain jopa hyperdimensionaalisia useissa reaaliulottuvuuksissa. Kummassakin tapauksessa hänen dimensionaalisuutensa tulee aivan jostain muualta kuin siitä, että hän tarvitsisi kompleksilukuja. Kron ei ole sanonut, että mitään ongelmaa ei voi ratkaista ilman häntä.

        Lisäksi Kron ei ole koskaan sanonut, että häntä tarvitaan 'johonkin' eli 'yhtään mihinkään'. Kyseiset sähköpiirilaskut helpottavat ongelman käsittelyä ja ratkaisemista. Eivätkä ne kuvaa sitä, mitä ongelman takana olevan todellisuuden pitäisi kaikkien mielestä olla. Myöskään Kron ei ole sanonut, että hänen ratkaisunsa ovat tosia, eli että niiden tarkkuus on suurempi kuin sellaisen ns. todellisuuden, missä puhutaan 3D-hiukkanen kerrallaan. Hänen juttunsa ovat niin vanhoja, että niitä ei edes pidä verrata QED:iin.

        B:
        "Reduktionismi on filosofinen käsite jota sovelletaan luonnontieteissä ja kaikki tieteen hypoteesinmuodostus on myös aina läpikotaisin filosofista metafysiikkaa vaikka se olisi matematiikan kielen avulla ilmaistua."

        Luonnontiede päätyy reduktionismiin kuitenkin myös tieteellistä reittiä pitkin. Ei-reduktionistinen maailma olisi jotain, missä ei-reduktionismi aidosti ilmenee.

        Jos on olemassa käsite ja vastakäsite, ja todellisuus on vain toista niistä, niin kukaan ei voi jättäytyä käsitteiden ulkopuolelle tehdessään todellisuuden kanssa tutkimusta. Jotta tieteessä olisi jokin ei-reduktionistinen ala, sen pitäisi vain tehdä ei-reduktionistinen malli. Mutta tätä mallia koskee sama todistustaakka, ja sama kritisoinnin taakka, että se on jotain, mikä on suoraan filosofian kirjasta. Huomaa myös, että jos sanoo ettei ole olemassa todellisuutta, missä on käsite tai sen vastakäsite, tai että näiden sijaan tulisi olla muita käsitteitä, niin joku alkaa kritisoimaan sinua filosofiksi. Ei siis ole mitään tieteelle annettavaa neuvoa, mikä päätyisi siihen, ettet tulisi tänne enää sanomaan sitä metafyysiseksi.

        Tieteen täytyy käsittää, mitä reduktionismi on. Tai sopia se jonkun kanssa, joka näitä sanoja keksii. Sen lisäksi tieteen täytyy käsittää, mitä se itse on suhteessa reduktionismiin kaikissa tapauksissa missä se tiede on tiedettä. Muutoin tiede ei voisi joskus tehdä jotain uutta, minkä jälkeen se kertoisi sinullekin, että todellisuus voi olla ei-reduktionistinen havaitulla tavalla ym..

        4


      • Anonyymi
        Anonyymi kirjoitti:

        Kun tehdään jokin MIT:in mainitsema suurempi komponentti, se on mahdollista siksi, että on löydetty Maxwellin yhtälöiden mukainen tapaus, missä todellisuus komponentissa on Maxwellin yhtälöiden tilana todella erikoisessa järjetyksessä, joka on pysyvä. Komponentin tuottaminen ei ole aina mahdollista, jos ei yhtään tutki asiaa Maxwellin yhtälöiden tasolla, koska vain Maxwellin yhtälöiden tasolla voi nähdä, onko se täyttynyt häiriöistä ja esim. termeistä, jotka eivät ole toivottuja nollia. Vain etsimällä ja mittaamalla sellaisia termejä voi tietää, että ne ovat nollia ja että nykyinen rakennelma tulee rakentumaan hyödylliseksi. Tekstissäsi mainitut suuret käytännön hyödyt tulevat yleensä siitä, että on tuhansia tai miljoonia piirin komponentteja. Vain yhdessä niillä on jokin kodinkoneen merkitys. Mutta se miten tähän hyödyllisyyteen on päästy, on joskus ollut se, että nämä komponentit on pystytty yksitellen tuottamaan ja antamaan kokoonpanon suunnittelijan käyttöön muodossa, joka on ideaaliseksi todettu Maxwellin tilana. Toisaalta esim. ideaalisen kondensaattorin pystyy myös tekemään tuurilla ja intuitiolla, koska siinä ideaalisuus toteutuu ottamalla homogeenista puhdasta ainetta ja tekemällä siitä geometrisesti sileän muotoisen ja suoran. Tälläinen aineen yksinkertaistus 3D-avaruudessa johtaa kyseiseen yksinkertaiseen Maxwell-tilaan, jota sitten saa käyttää helpompien sääntöjen mukaan.

        Kun yllä kysytiin, että oletatko elektronien leviävän kaasuna virtapiiriin, niin tämä on jollain tavalla oletus, joka on ristiriidassa sen kanssa, mitä virtapiirilait olettavat olevan mahdollista siinä aineessa, mitä ne olettavat virtapiirin olevan. Eli sinun oletuksesi mukaan nämä lait pitäisi poistaa, ja uudet lait tehtäisiin sen mukaan mitä materiaalissa esim. kuvittelee tai näkee olevan, minkä jälkeen siitä kirjoitetaan jonkinlainen Maxwellin yhtälöiden mukainen elektronien ja niiden ympäristön välinen liikkumisen muoto.

        Kron ei ole sanonut koskaan, että avaruus on kompleksiavaruus. Sen sijaan hän on käyttänyt 3D-avaruudessa (tai kokonaan avaruuden ulkopuolella, koska usein laskuissa ei ole tarkoitus edetä avaruudessa) sellaisia objekteja, jotka ovat kompleksisia. Objektit, joita hän käytti eivät ole aina kompleksisia, joten ei ole totta, että Kron edellyttää kaikkien hänen sähkökohteidensa olevan ongelmassa kompleksilukujen muodossa. Lisäksi Kron on tehnyt ongelmista varmaan monen dimension objekteja, joiden avaruus on kuitenkin tavallinen. Tai sitten hän on lisäksi tehnyt jotain jopa hyperdimensionaalisia useissa reaaliulottuvuuksissa. Kummassakin tapauksessa hänen dimensionaalisuutensa tulee aivan jostain muualta kuin siitä, että hän tarvitsisi kompleksilukuja. Kron ei ole sanonut, että mitään ongelmaa ei voi ratkaista ilman häntä.

        Lisäksi Kron ei ole koskaan sanonut, että häntä tarvitaan 'johonkin' eli 'yhtään mihinkään'. Kyseiset sähköpiirilaskut helpottavat ongelman käsittelyä ja ratkaisemista. Eivätkä ne kuvaa sitä, mitä ongelman takana olevan todellisuuden pitäisi kaikkien mielestä olla. Myöskään Kron ei ole sanonut, että hänen ratkaisunsa ovat tosia, eli että niiden tarkkuus on suurempi kuin sellaisen ns. todellisuuden, missä puhutaan 3D-hiukkanen kerrallaan. Hänen juttunsa ovat niin vanhoja, että niitä ei edes pidä verrata QED:iin.

        B:
        "Reduktionismi on filosofinen käsite jota sovelletaan luonnontieteissä ja kaikki tieteen hypoteesinmuodostus on myös aina läpikotaisin filosofista metafysiikkaa vaikka se olisi matematiikan kielen avulla ilmaistua."

        Luonnontiede päätyy reduktionismiin kuitenkin myös tieteellistä reittiä pitkin. Ei-reduktionistinen maailma olisi jotain, missä ei-reduktionismi aidosti ilmenee.

        Jos on olemassa käsite ja vastakäsite, ja todellisuus on vain toista niistä, niin kukaan ei voi jättäytyä käsitteiden ulkopuolelle tehdessään todellisuuden kanssa tutkimusta. Jotta tieteessä olisi jokin ei-reduktionistinen ala, sen pitäisi vain tehdä ei-reduktionistinen malli. Mutta tätä mallia koskee sama todistustaakka, ja sama kritisoinnin taakka, että se on jotain, mikä on suoraan filosofian kirjasta. Huomaa myös, että jos sanoo ettei ole olemassa todellisuutta, missä on käsite tai sen vastakäsite, tai että näiden sijaan tulisi olla muita käsitteitä, niin joku alkaa kritisoimaan sinua filosofiksi. Ei siis ole mitään tieteelle annettavaa neuvoa, mikä päätyisi siihen, ettet tulisi tänne enää sanomaan sitä metafyysiseksi.

        Tieteen täytyy käsittää, mitä reduktionismi on. Tai sopia se jonkun kanssa, joka näitä sanoja keksii. Sen lisäksi tieteen täytyy käsittää, mitä se itse on suhteessa reduktionismiin kaikissa tapauksissa missä se tiede on tiedettä. Muutoin tiede ei voisi joskus tehdä jotain uutta, minkä jälkeen se kertoisi sinullekin, että todellisuus voi olla ei-reduktionistinen havaitulla tavalla ym..

        4

        Ehkä se kertoo enemmän filosofiasta, että filosofian löytää monesta asiasta. Tähän liittyen matematiikan kieli ei ole erityisesti jotain filosofia-vapaata aluetta. Lisäksi on mahdollista, että todellisuus on matemaattinen, jolloin kyseessä ei ole minkään asian ilmaisu vaan sellainen luonnontiede olisi aivan todellisuuden ytimessä ja ydin olisi kerrottu. Matematiikassa on muuten olemassa eri tasoja siten, että jokin asia on matematiikkaa, ja tällä asialla on monta matemaattista esitystä, jotka ovat omaa matematiikkaansa ja tekevät kuin toisen tason.

        Filosofiassa on kuitenkin paljon menetelmiä, mitä ei ole mitään syytä käyttää tieteessä tai matematiikassa ja siksi näistä ei kannata etsiä yhteisiä puolia joka viestissä.

        Et sanonut suoraan, että reduktionismi on metafysiikkaa. Mutta et koskaan mainitse mitään varsinaista muutakaan filosofiaan liittyvää asiaa, mikä tieteessä esiintyisi. Metafysiikan määritelmä on olla jotain, missä ei ole kyse enää fysiikasta. Tämä on parempi määritelmä kuin sanoa, että kaikki mitä ei havaita, on metafyysistä. Koska silloin tässä havaitsemattomassa asiassa voi olla sen näkymättömyyden lisäksi sama metafyysinen kysymysmäärä, joka esiintyy myös monessa havaitussa asiassa, ja se esiintyisi erikseen fyysisten kysymysten lisäksi. Jos fysiikka täyttyy asioista, joita ei tiedetä eikä havaita, nämä voivat olla esim. filosofisia kysymyksiä, mutta jos lukisit joskus näitä ja metafysiikkaa, näkisit ettei niissä puhuta samasta asiasta. Empiirinen tiede, jossa ei ole filosofisia kysymyksiä yhtään on joskus myös nimeltään naturalismia. Olet monesti tehnyt viestejä, joissa sanotaan tieteen olevan naturalismia ja teet niistä lopulta vain ristiriitaisen väitteen. (Tässä viestissä itse kannattamiasi asioita on naturalismi eli kaikki kokeiltu ja sovellettu asia.)

        Havaitsemattomista asioista puhuminen voi sinusta olla yhtä vastenmielinen asia sekä tieteessä, että jossain, missä uskotaan havaitsemattomiin entiteetteihin ja seurataan niitä. Tällöin sinun tekemäsi yhtäläistäminen näiden asioiden välille ei ole ehkä filosofinen ja tarkoitettu seuraamaan samoja linjoja kuin olen esittänyt, vaan sekin on jotain, mikä tulee puhtaasti käytännöstä ja toimivuudesta. Jos et halua puhua siitä, miten tieteen pitäisi keskittyä havaintoihin filosofiselta kannalta, niin sinun ei varmaan kannata käyttää filosofian terminologiaa ohjatessasi tieteen suuntaa. Voisit sanoa vain, että on väärin, kun on näkymätöntä ja käyttää niitä esimerkkejä entiteeteistä kuin oikeasti haluat. Jotkut eivät ehkä näe samaa yhtäläisyyttä kuin sinä näet, ja jotkut näkevät sen jo liiankin hyvin, eivätkä varsinaisesti ajattele niin, että oli hyvä, kun muistakin näkymättömistä vaihtoehdoista piti mainita.

        Väitän siis lisäksi, että reduktionismi, mikä tieteeseen liittyy ei ole metafysiikkaa. Tiede tekee sitä aina tieteellisesti esitettynä asiana, tai se on vain yhden mallin ominaisuus, josta on vain tieteellisiä ajatuksia seurauksena, koska mallilla, missä sitä on, ei edes pyritä muuhun. Lisäksi metafysiikkaa ei tule paljon mistään muustakaan. Kun sillä hetkellä, kun fyysikko sanoo jotain jostakin, siitä asiasta tulee pelkkää fysiikkaa, ja vain fysiikka-asioihin sitä voi siitä lähtien käyttää. Jokin havaitsematon asia, olisi sen havaitsemisen tapauksessa tai 'siellä oltaessa' pelkkää fysiikkaa. Tässä ei ole merkitystä kuka sen sanoo ja miten tarkasti, vaan tarkoitan että suurin osa ihmisten vääristäkin mielikuvista on aina suureksi osaksi fysiikaksi ajateltuja asioita ja fysiikan kysymyksiä.

        B:
        "Fyysikot harrastavat matemaattista taskubiljardia jos yriittävät hahmottaa todellisuutta matematiikka edellä ja filosofit harrastavat hyvin samantapaista käsiteakrobatiaa pelkkien mielikuvien tasolla ja kummallakaan lähestymistavalla ei ole välttämättä ole juuri mitään tekemistä todellisuuden kanssa."

        Olet keksinyt nämä lähestymistavat kokonaan itse etkä yhtään ottanut huomioon oikeiden ihmisten oikeita lähestymistapoja. Jos saat päähäsi ajatuksen, että joku tekee jotain matemaattista eikä ota siinä todellisuutta huomioon, niin kritisoit vain omia ajatuksiasi ('ajatus' tässä ei tarkoita päätelmää, tai että päätät jotain, vaan ajatukset ovat pikemminkin sinua hallitsevia asioita).

        5


      • Anonyymi
        Anonyymi kirjoitti:

        Ehkä se kertoo enemmän filosofiasta, että filosofian löytää monesta asiasta. Tähän liittyen matematiikan kieli ei ole erityisesti jotain filosofia-vapaata aluetta. Lisäksi on mahdollista, että todellisuus on matemaattinen, jolloin kyseessä ei ole minkään asian ilmaisu vaan sellainen luonnontiede olisi aivan todellisuuden ytimessä ja ydin olisi kerrottu. Matematiikassa on muuten olemassa eri tasoja siten, että jokin asia on matematiikkaa, ja tällä asialla on monta matemaattista esitystä, jotka ovat omaa matematiikkaansa ja tekevät kuin toisen tason.

        Filosofiassa on kuitenkin paljon menetelmiä, mitä ei ole mitään syytä käyttää tieteessä tai matematiikassa ja siksi näistä ei kannata etsiä yhteisiä puolia joka viestissä.

        Et sanonut suoraan, että reduktionismi on metafysiikkaa. Mutta et koskaan mainitse mitään varsinaista muutakaan filosofiaan liittyvää asiaa, mikä tieteessä esiintyisi. Metafysiikan määritelmä on olla jotain, missä ei ole kyse enää fysiikasta. Tämä on parempi määritelmä kuin sanoa, että kaikki mitä ei havaita, on metafyysistä. Koska silloin tässä havaitsemattomassa asiassa voi olla sen näkymättömyyden lisäksi sama metafyysinen kysymysmäärä, joka esiintyy myös monessa havaitussa asiassa, ja se esiintyisi erikseen fyysisten kysymysten lisäksi. Jos fysiikka täyttyy asioista, joita ei tiedetä eikä havaita, nämä voivat olla esim. filosofisia kysymyksiä, mutta jos lukisit joskus näitä ja metafysiikkaa, näkisit ettei niissä puhuta samasta asiasta. Empiirinen tiede, jossa ei ole filosofisia kysymyksiä yhtään on joskus myös nimeltään naturalismia. Olet monesti tehnyt viestejä, joissa sanotaan tieteen olevan naturalismia ja teet niistä lopulta vain ristiriitaisen väitteen. (Tässä viestissä itse kannattamiasi asioita on naturalismi eli kaikki kokeiltu ja sovellettu asia.)

        Havaitsemattomista asioista puhuminen voi sinusta olla yhtä vastenmielinen asia sekä tieteessä, että jossain, missä uskotaan havaitsemattomiin entiteetteihin ja seurataan niitä. Tällöin sinun tekemäsi yhtäläistäminen näiden asioiden välille ei ole ehkä filosofinen ja tarkoitettu seuraamaan samoja linjoja kuin olen esittänyt, vaan sekin on jotain, mikä tulee puhtaasti käytännöstä ja toimivuudesta. Jos et halua puhua siitä, miten tieteen pitäisi keskittyä havaintoihin filosofiselta kannalta, niin sinun ei varmaan kannata käyttää filosofian terminologiaa ohjatessasi tieteen suuntaa. Voisit sanoa vain, että on väärin, kun on näkymätöntä ja käyttää niitä esimerkkejä entiteeteistä kuin oikeasti haluat. Jotkut eivät ehkä näe samaa yhtäläisyyttä kuin sinä näet, ja jotkut näkevät sen jo liiankin hyvin, eivätkä varsinaisesti ajattele niin, että oli hyvä, kun muistakin näkymättömistä vaihtoehdoista piti mainita.

        Väitän siis lisäksi, että reduktionismi, mikä tieteeseen liittyy ei ole metafysiikkaa. Tiede tekee sitä aina tieteellisesti esitettynä asiana, tai se on vain yhden mallin ominaisuus, josta on vain tieteellisiä ajatuksia seurauksena, koska mallilla, missä sitä on, ei edes pyritä muuhun. Lisäksi metafysiikkaa ei tule paljon mistään muustakaan. Kun sillä hetkellä, kun fyysikko sanoo jotain jostakin, siitä asiasta tulee pelkkää fysiikkaa, ja vain fysiikka-asioihin sitä voi siitä lähtien käyttää. Jokin havaitsematon asia, olisi sen havaitsemisen tapauksessa tai 'siellä oltaessa' pelkkää fysiikkaa. Tässä ei ole merkitystä kuka sen sanoo ja miten tarkasti, vaan tarkoitan että suurin osa ihmisten vääristäkin mielikuvista on aina suureksi osaksi fysiikaksi ajateltuja asioita ja fysiikan kysymyksiä.

        B:
        "Fyysikot harrastavat matemaattista taskubiljardia jos yriittävät hahmottaa todellisuutta matematiikka edellä ja filosofit harrastavat hyvin samantapaista käsiteakrobatiaa pelkkien mielikuvien tasolla ja kummallakaan lähestymistavalla ei ole välttämättä ole juuri mitään tekemistä todellisuuden kanssa."

        Olet keksinyt nämä lähestymistavat kokonaan itse etkä yhtään ottanut huomioon oikeiden ihmisten oikeita lähestymistapoja. Jos saat päähäsi ajatuksen, että joku tekee jotain matemaattista eikä ota siinä todellisuutta huomioon, niin kritisoit vain omia ajatuksiasi ('ajatus' tässä ei tarkoita päätelmää, tai että päätät jotain, vaan ajatukset ovat pikemminkin sinua hallitsevia asioita).

        5

        B:
        "Kokeellinen tiede ja tekniikan kehitys taas pakottaa käytännön tasolle jossa matematiikka ja kielen käsitteet ovat pelkästään apuvälineitä eikä mikään itsetarkoitus. Toimivuus eli käytännön tason sovellettavuus ovat olennaisin asia eikä teoreettiset loogis-matemaattiset konstruktiot jotka palvelevat ainoastaan kommunikoinnin tasolla."

        Kielen käsite nimeltä 'asioiden itsetarkoitus' -sellaisen löytäminen ja tunnistaminen - sellaisessa projektissa kuin käytännön tiede, on vähän sama kuin sen johtajan (poliittiset rahoittajat) tai konsulttien (tieteenfilosofit) pitämä juhlapuhe, tai ohjaava lausunto. Oikeat tekijät joutuvat matematiikan kanssa tekemisiin jatkuvasti. He joutuisivat varmaan myös kielen kanssa tekemisiin yhtä paljon, jos on joskus oltava sillä tavalla, että kieli kertoisi kaiken, mitä todellisuudessa on, ja jättämättä mitään pois. Mutta kieleksi riittää lähinnä matematiikka.

        Tieteessä ei ole mitään asiaa (varsinkaan sovellusta), mikä pitää saada toimimaan. Jos jokin asia ei toimi, voi olla syynä se, että todellisuudessa sen ei pidä toimia. Pikemminkin kaikki mitä on, toimii jo, ja sitä pitää vain selittää. Jos pitää. Samalla voi olla, että vain noin 0.01 % todellisuudesta (jollain keksityllä tavalla laskettuna) on sovellettavissa johonkin, mikä pitää ns. tieteellisesti tehdä sovellukseksi. Esim. kaikellle kuparimalmille, mitä ei ole jalostettu puhtaiksi langoiksi, ei ole keksitty muuta nimeä kuin jäte.

        Jos haluat aloittaa väitelmän siitä, mikä on tieteen tarkoitus, niin ensimmäisenä asiana nousee esiin se, että miksi tämä väitelmä pitää käydä, ja miksi pitää tietää tieteen tarkoitus? Sanoisin että tieteen tarkoitus on tietämisessä, mihin liittyy esim. se että vastakohtana olevasta tietämättömyydestä voi olla käytännönkin haittaa. Sitä ei tulisi sanoa minusta kommunikoinnin tasoksi. Voit verrata siihen, että onko sinusta tieteen tarkoituksen tietäminen ja uudelleen määrittely vain kommunikoinnin tasoa. Kommunikoinnin taso on myös todella käytännöllinen, joten jos parantaa kommunikoinnin tasoa, niin luulisi aina tekevän jotain käytännössä hyödyllistä.

        B:
        "Atomien, molekyylien, elinten ja eliöiden tasot vaikuttavat kaikki rinnakkain samanaikaisesti..."

        Ei ole näyttöä. Esim. voisit aloittaa muutamasta molekyylistä ja sanoa, mitä ne tekevät rinnakkain atomien tekemisen kanssa. Molekyylit voisivat näkyä tosi hyvin laitteilla tämän tarkistamiseksi.

        B:
        "...mutta jos eliön keho on terve ja sillä on jonkintasoista kognitiota (kuten jopa yksisoluisilla tohvelieläimillä on todettu) niin sen eliön molekyylit, atomit ja atomien sisäiset entiteetit liikkuvat sinne minne se kognitio kulloinkin ohjaa eikä mikään fysiikan matemaattinen kaava voi kertoi missä ja miten se tapahtuu ja sama koskee vielä suuremmassa määrin monisoluisia kuten ihmisiä jotka eivät ole mitenkään tarkkaanrajattuja kokonaisuuksia edes elinten ja solujen tasolla vaan säilyttävät kokonaisidentieettinsä aktiivisina toimijoina niin kauan kuin heidän kognitionsa on aktiivisessa yhteydessä heidän kehoonsa eli valveilla ollessa. "

        Mikään kognition selitys ei voi kertoa, missä ja miten tapahtuu tohvelieläimen koko kehon liike. Pidän sitä jotenkin helpompana asiana selittää, vaikka keho on kaiken järjen mukaan monimutkaisempi asia kuin yksi atomi.

        Mikään kognition selitys ei voi kertoa, missä ja miten tapahtuu tohvelielämen sisällä olevan atomin liike.

        Fysiikan kaava pystyy ja on aina jokaisesta kaavaversiosta puhuttaessa pystynyt teoriassa kertomaan tohveliatomin tapahtumat. Niitä kaavoja ei ole koskaan käytetty kyseiseen ongelmaan. Ei siis ole mitään näyttöä mistään asiasta, mitä ne eivät 'voi' tehdä.

        Kognition selitys ei voi teoriassa koskaan kertoa, mitä atomille tulee tapahtumaan. Koska 'kognition selitys' ei ole mitään muuta kuin minun (ei edes B:n) tekemäni sanapari. Sanaparista ei seuraa mitään tietoa.

        Tohvelieläimen kognitio on keksitty todennäköisesti aineen perusteella siten, että otetaan eläin ja pommitetaan sitä aineella, joka on sille aiemmin makroskooppisesti nähtynä biologisesti tuttua. Eläimen ainetta mitataan ja nähdään, että aine eläimessä reagoi pommitukseen aika säännöllisellä tavalla, joka muistuttaa varmaan paljon kuparijohtoja, joissa ei mene kaasua. Sitten toh velieläimestä löytyy pitkän matkan päästä atomi, ja atomi kulkee kohti pommitettua paikkaa. Joku joka ei tunne kaikkea ainetta ja sen liikettä johtojen ja tämän atomin väliltä sanoisi, että välissä oli kognitio, mutta yhtä hyvin atomi on osa samaa virtapiiriä, missä pommitus saa piirin tuomaan toisen piirin osan lähemmäs.

        6


      • Anonyymi
        Anonyymi kirjoitti:

        B:
        "Kokeellinen tiede ja tekniikan kehitys taas pakottaa käytännön tasolle jossa matematiikka ja kielen käsitteet ovat pelkästään apuvälineitä eikä mikään itsetarkoitus. Toimivuus eli käytännön tason sovellettavuus ovat olennaisin asia eikä teoreettiset loogis-matemaattiset konstruktiot jotka palvelevat ainoastaan kommunikoinnin tasolla."

        Kielen käsite nimeltä 'asioiden itsetarkoitus' -sellaisen löytäminen ja tunnistaminen - sellaisessa projektissa kuin käytännön tiede, on vähän sama kuin sen johtajan (poliittiset rahoittajat) tai konsulttien (tieteenfilosofit) pitämä juhlapuhe, tai ohjaava lausunto. Oikeat tekijät joutuvat matematiikan kanssa tekemisiin jatkuvasti. He joutuisivat varmaan myös kielen kanssa tekemisiin yhtä paljon, jos on joskus oltava sillä tavalla, että kieli kertoisi kaiken, mitä todellisuudessa on, ja jättämättä mitään pois. Mutta kieleksi riittää lähinnä matematiikka.

        Tieteessä ei ole mitään asiaa (varsinkaan sovellusta), mikä pitää saada toimimaan. Jos jokin asia ei toimi, voi olla syynä se, että todellisuudessa sen ei pidä toimia. Pikemminkin kaikki mitä on, toimii jo, ja sitä pitää vain selittää. Jos pitää. Samalla voi olla, että vain noin 0.01 % todellisuudesta (jollain keksityllä tavalla laskettuna) on sovellettavissa johonkin, mikä pitää ns. tieteellisesti tehdä sovellukseksi. Esim. kaikellle kuparimalmille, mitä ei ole jalostettu puhtaiksi langoiksi, ei ole keksitty muuta nimeä kuin jäte.

        Jos haluat aloittaa väitelmän siitä, mikä on tieteen tarkoitus, niin ensimmäisenä asiana nousee esiin se, että miksi tämä väitelmä pitää käydä, ja miksi pitää tietää tieteen tarkoitus? Sanoisin että tieteen tarkoitus on tietämisessä, mihin liittyy esim. se että vastakohtana olevasta tietämättömyydestä voi olla käytännönkin haittaa. Sitä ei tulisi sanoa minusta kommunikoinnin tasoksi. Voit verrata siihen, että onko sinusta tieteen tarkoituksen tietäminen ja uudelleen määrittely vain kommunikoinnin tasoa. Kommunikoinnin taso on myös todella käytännöllinen, joten jos parantaa kommunikoinnin tasoa, niin luulisi aina tekevän jotain käytännössä hyödyllistä.

        B:
        "Atomien, molekyylien, elinten ja eliöiden tasot vaikuttavat kaikki rinnakkain samanaikaisesti..."

        Ei ole näyttöä. Esim. voisit aloittaa muutamasta molekyylistä ja sanoa, mitä ne tekevät rinnakkain atomien tekemisen kanssa. Molekyylit voisivat näkyä tosi hyvin laitteilla tämän tarkistamiseksi.

        B:
        "...mutta jos eliön keho on terve ja sillä on jonkintasoista kognitiota (kuten jopa yksisoluisilla tohvelieläimillä on todettu) niin sen eliön molekyylit, atomit ja atomien sisäiset entiteetit liikkuvat sinne minne se kognitio kulloinkin ohjaa eikä mikään fysiikan matemaattinen kaava voi kertoi missä ja miten se tapahtuu ja sama koskee vielä suuremmassa määrin monisoluisia kuten ihmisiä jotka eivät ole mitenkään tarkkaanrajattuja kokonaisuuksia edes elinten ja solujen tasolla vaan säilyttävät kokonaisidentieettinsä aktiivisina toimijoina niin kauan kuin heidän kognitionsa on aktiivisessa yhteydessä heidän kehoonsa eli valveilla ollessa. "

        Mikään kognition selitys ei voi kertoa, missä ja miten tapahtuu tohvelieläimen koko kehon liike. Pidän sitä jotenkin helpompana asiana selittää, vaikka keho on kaiken järjen mukaan monimutkaisempi asia kuin yksi atomi.

        Mikään kognition selitys ei voi kertoa, missä ja miten tapahtuu tohvelielämen sisällä olevan atomin liike.

        Fysiikan kaava pystyy ja on aina jokaisesta kaavaversiosta puhuttaessa pystynyt teoriassa kertomaan tohveliatomin tapahtumat. Niitä kaavoja ei ole koskaan käytetty kyseiseen ongelmaan. Ei siis ole mitään näyttöä mistään asiasta, mitä ne eivät 'voi' tehdä.

        Kognition selitys ei voi teoriassa koskaan kertoa, mitä atomille tulee tapahtumaan. Koska 'kognition selitys' ei ole mitään muuta kuin minun (ei edes B:n) tekemäni sanapari. Sanaparista ei seuraa mitään tietoa.

        Tohvelieläimen kognitio on keksitty todennäköisesti aineen perusteella siten, että otetaan eläin ja pommitetaan sitä aineella, joka on sille aiemmin makroskooppisesti nähtynä biologisesti tuttua. Eläimen ainetta mitataan ja nähdään, että aine eläimessä reagoi pommitukseen aika säännöllisellä tavalla, joka muistuttaa varmaan paljon kuparijohtoja, joissa ei mene kaasua. Sitten toh velieläimestä löytyy pitkän matkan päästä atomi, ja atomi kulkee kohti pommitettua paikkaa. Joku joka ei tunne kaikkea ainetta ja sen liikettä johtojen ja tämän atomin väliltä sanoisi, että välissä oli kognitio, mutta yhtä hyvin atomi on osa samaa virtapiiriä, missä pommitus saa piirin tuomaan toisen piirin osan lähemmäs.

        6

        Kognition määritelmä muuna kuin hauskana ilmauksena piirien tapahtumille ovat hankalia, koska jos sanoo mikä kognitio on, tästä tarvitaan mieluiten todistus, että se on jokin kohde, joka on olemassa. Jos sanoo, että kognitio on se, mikä siirtää atomeita, tässä on monta asiaa, mitä yritetään kommunikoinnin tasolla nimetä uudestaan toisella nimellä, eikä paljon muuta. Jos kognitio on se, mikä siirtää atomeita, kun toiset eivät siirrä niitä, on sama kuin ei uskoisi vielä tässäkään vaiheessa, että tiede pystyy tekemään mistä tahansa atomisiirrosta oman sääntönsä (missä jos vielä tarkkaavaisia ollaan niin '...kun toiset eivät siirrä' muuttuu aina tyhjäksi joukoksi).

        B:
        "Korkeamman organisaatiotason kokonaisuuksia ei kykene kunnolla edes hahmottamaan pelkästään mikroskoopin mittakaavassa..."

        Organisaation tason korkeus on määritelty väärin eikä niitä ole olemassa.

        Todellisuus ei myöskään muotodu sen mukaan, mitä kykenee hahmottamaan, joten se mitä tässä lukee ei ole tieteellinen lause.

        Oletko koskaan nähnyt teleskooppimittakaavassa hahmotettua universumia, kun teleskooppi näkee siitä suhteessa ns. mikroskooppisen osan? (Vastaus on ei.) Onko hahmotus universumista parempi ilman teleskooppia? Tässä on tarkoitus verrata siihen, että käyttää teleskoopia kerran, jotta löytyy käsite nimeltä galaksi. Sitten alat käyttää käsitettä galaksi, ja teet mielikuvituksellasi universumin, missä käsitettä galaksi saa käyttää vapaasti ja esim. kognitio-käsitteen yhteydessä.

        Tieteessä tehdään monien osien objekteista joskus suuren datan tutkimusta, missä jokaista osaa mitataan, ja sen jälkeen datat tutkitaan yhdessä suureen dataan tarkoitetulla matemaattisella menetelmällä. Se kykenee yksittäisiin osiin ja suureen määrään niitä, sekä niiden laajojen efektien tutkimiseen. Tai ainakin jos on olemassa joku, joka kykenee hahmottamaan jotain suuressa koossa, niin jos tämä antaa oman mittauksensa käyttöön, niin sitä voi verrata osien dataan, joka on samanaikaista.

        B:
        "Kiinteä aine ei kykene itseorgaisoitumaan vaikka nesteet ja kaasut siihen kykenevät joissain olosuhteissa kuten esim. cymatics ilmiössä äänen resonanssien kautta. Samantapainen magneettinen harmooninen resonanssi lienee kaiken mahdollisen itseorganisoitumisen taustalla joka selittänee kaiken itseorganisoitumisen atomeista galaksien tasolle."

        Kiinteä aine on jo organisoituneena tosi paljon ja sitä myös yleensä esiintyy saavutetuna organisaationa.

        Cymatic-ilmiössä hiekka on kiinteää ainetta. Samoin se levy, jolla hiekkaa on. Ne eivät ole itseorganisoituvia. Cymatiikka on aivan sama organisointiasia kuin piirtäisi hiekkaan sanoja. Siinä sormet on vain korvailtu soittimilla, ilmakehällä ja levyllä. Samoin magneettikentät olisivat vain sormien korvaaja, eikä kyseessä ole itseorganisointi vaan pakollinen organisointi.

        Atomi on organisoitunut tila verrattuna satunnaiseen elektroniin ja protoniin laatikossa. Tiede on tutkinut atomien muodostumisen syyt ja katsonut paikalla olevia magneettikenttiä tarkkaavaisesti. Itseorganisaatiossa ei tutkita yhtään asiaa, joka vain sijaitsee radallaan potentiaalikuilun luona. Aine, joka kiertää potentiaalikuilua esim. Newtonin liikeyhtälön mukaan on entropialtaan nolla. Kiinteä aine tekee niin ja sen organisaatio on jo tietyssä maksimissa. Samoin tekevät galaksin yksittäiset pistemäiset kiertolaiset, (ei kaasu) jotka ovat yhtä kuolleita kuin pysyvästi kristalloitunut kiinteä aine, kun ei katso niihin mikroskoopilla.

        Itseorganisointi-asioissa kuitenkin joskus tutkitaan liian vähän asioita, kuten aineessa suositut taajuudet ja mihin ne johtavat. Joten muissa aineissa voisi ajatella, että mikä tahansa sellainen voi estää ergodisuutta ja alkaa tekemään yhdestä kohteesta vähemmän entrooppista. Kun on olemassa sekoitus aineita, jotka eivät enää muutu toisikseen, mutta jotka ovat samanlaisia keskenään (eikä esim. atomit vs. valo) se voi myös ikäänkuin valita nimenomaisen aineen, joka on etnropiassa alin muiden entropian kasvaessa.

        7


      • Anonyymi
        Anonyymi kirjoitti:

        Kognition määritelmä muuna kuin hauskana ilmauksena piirien tapahtumille ovat hankalia, koska jos sanoo mikä kognitio on, tästä tarvitaan mieluiten todistus, että se on jokin kohde, joka on olemassa. Jos sanoo, että kognitio on se, mikä siirtää atomeita, tässä on monta asiaa, mitä yritetään kommunikoinnin tasolla nimetä uudestaan toisella nimellä, eikä paljon muuta. Jos kognitio on se, mikä siirtää atomeita, kun toiset eivät siirrä niitä, on sama kuin ei uskoisi vielä tässäkään vaiheessa, että tiede pystyy tekemään mistä tahansa atomisiirrosta oman sääntönsä (missä jos vielä tarkkaavaisia ollaan niin '...kun toiset eivät siirrä' muuttuu aina tyhjäksi joukoksi).

        B:
        "Korkeamman organisaatiotason kokonaisuuksia ei kykene kunnolla edes hahmottamaan pelkästään mikroskoopin mittakaavassa..."

        Organisaation tason korkeus on määritelty väärin eikä niitä ole olemassa.

        Todellisuus ei myöskään muotodu sen mukaan, mitä kykenee hahmottamaan, joten se mitä tässä lukee ei ole tieteellinen lause.

        Oletko koskaan nähnyt teleskooppimittakaavassa hahmotettua universumia, kun teleskooppi näkee siitä suhteessa ns. mikroskooppisen osan? (Vastaus on ei.) Onko hahmotus universumista parempi ilman teleskooppia? Tässä on tarkoitus verrata siihen, että käyttää teleskoopia kerran, jotta löytyy käsite nimeltä galaksi. Sitten alat käyttää käsitettä galaksi, ja teet mielikuvituksellasi universumin, missä käsitettä galaksi saa käyttää vapaasti ja esim. kognitio-käsitteen yhteydessä.

        Tieteessä tehdään monien osien objekteista joskus suuren datan tutkimusta, missä jokaista osaa mitataan, ja sen jälkeen datat tutkitaan yhdessä suureen dataan tarkoitetulla matemaattisella menetelmällä. Se kykenee yksittäisiin osiin ja suureen määrään niitä, sekä niiden laajojen efektien tutkimiseen. Tai ainakin jos on olemassa joku, joka kykenee hahmottamaan jotain suuressa koossa, niin jos tämä antaa oman mittauksensa käyttöön, niin sitä voi verrata osien dataan, joka on samanaikaista.

        B:
        "Kiinteä aine ei kykene itseorgaisoitumaan vaikka nesteet ja kaasut siihen kykenevät joissain olosuhteissa kuten esim. cymatics ilmiössä äänen resonanssien kautta. Samantapainen magneettinen harmooninen resonanssi lienee kaiken mahdollisen itseorganisoitumisen taustalla joka selittänee kaiken itseorganisoitumisen atomeista galaksien tasolle."

        Kiinteä aine on jo organisoituneena tosi paljon ja sitä myös yleensä esiintyy saavutetuna organisaationa.

        Cymatic-ilmiössä hiekka on kiinteää ainetta. Samoin se levy, jolla hiekkaa on. Ne eivät ole itseorganisoituvia. Cymatiikka on aivan sama organisointiasia kuin piirtäisi hiekkaan sanoja. Siinä sormet on vain korvailtu soittimilla, ilmakehällä ja levyllä. Samoin magneettikentät olisivat vain sormien korvaaja, eikä kyseessä ole itseorganisointi vaan pakollinen organisointi.

        Atomi on organisoitunut tila verrattuna satunnaiseen elektroniin ja protoniin laatikossa. Tiede on tutkinut atomien muodostumisen syyt ja katsonut paikalla olevia magneettikenttiä tarkkaavaisesti. Itseorganisaatiossa ei tutkita yhtään asiaa, joka vain sijaitsee radallaan potentiaalikuilun luona. Aine, joka kiertää potentiaalikuilua esim. Newtonin liikeyhtälön mukaan on entropialtaan nolla. Kiinteä aine tekee niin ja sen organisaatio on jo tietyssä maksimissa. Samoin tekevät galaksin yksittäiset pistemäiset kiertolaiset, (ei kaasu) jotka ovat yhtä kuolleita kuin pysyvästi kristalloitunut kiinteä aine, kun ei katso niihin mikroskoopilla.

        Itseorganisointi-asioissa kuitenkin joskus tutkitaan liian vähän asioita, kuten aineessa suositut taajuudet ja mihin ne johtavat. Joten muissa aineissa voisi ajatella, että mikä tahansa sellainen voi estää ergodisuutta ja alkaa tekemään yhdestä kohteesta vähemmän entrooppista. Kun on olemassa sekoitus aineita, jotka eivät enää muutu toisikseen, mutta jotka ovat samanlaisia keskenään (eikä esim. atomit vs. valo) se voi myös ikäänkuin valita nimenomaisen aineen, joka on etnropiassa alin muiden entropian kasvaessa.

        7

        Vain seitsemän kommenttia! Ennätys taitaa olla tähän mennessä 13.

        Täytyy jossain vaiheessa yrittää kirjoittaa niin provosoivasti että onnistut ylittämään kahdenkymmenen kommentin rajan yhteen omaan viestiini . :D

        Tuotoksesi ovat muuten erinomainen esimerkki Gurdjieffin kuvaileman formatoorisen apparaatin toiminnasta joka kykenee pelkästään systemaattiseen itsepetokseen varsinkin jos intuitiiviset kyvyt ovat täysin surkastuneet pois muodollisen koulutuksen seurauksena.

        Tuo sinun vitsisi alkaa kyllä käydä aika vanhaksi ja puuduttavaksi eikä enää toimi edes tahattoman huumorin ja viihteen tasolla.

        B


      • Anonyymi
        Anonyymi kirjoitti:

        B:
        "Kokeellinen tiede ja tekniikan kehitys taas pakottaa käytännön tasolle jossa matematiikka ja kielen käsitteet ovat pelkästään apuvälineitä eikä mikään itsetarkoitus. Toimivuus eli käytännön tason sovellettavuus ovat olennaisin asia eikä teoreettiset loogis-matemaattiset konstruktiot jotka palvelevat ainoastaan kommunikoinnin tasolla."

        Kielen käsite nimeltä 'asioiden itsetarkoitus' -sellaisen löytäminen ja tunnistaminen - sellaisessa projektissa kuin käytännön tiede, on vähän sama kuin sen johtajan (poliittiset rahoittajat) tai konsulttien (tieteenfilosofit) pitämä juhlapuhe, tai ohjaava lausunto. Oikeat tekijät joutuvat matematiikan kanssa tekemisiin jatkuvasti. He joutuisivat varmaan myös kielen kanssa tekemisiin yhtä paljon, jos on joskus oltava sillä tavalla, että kieli kertoisi kaiken, mitä todellisuudessa on, ja jättämättä mitään pois. Mutta kieleksi riittää lähinnä matematiikka.

        Tieteessä ei ole mitään asiaa (varsinkaan sovellusta), mikä pitää saada toimimaan. Jos jokin asia ei toimi, voi olla syynä se, että todellisuudessa sen ei pidä toimia. Pikemminkin kaikki mitä on, toimii jo, ja sitä pitää vain selittää. Jos pitää. Samalla voi olla, että vain noin 0.01 % todellisuudesta (jollain keksityllä tavalla laskettuna) on sovellettavissa johonkin, mikä pitää ns. tieteellisesti tehdä sovellukseksi. Esim. kaikellle kuparimalmille, mitä ei ole jalostettu puhtaiksi langoiksi, ei ole keksitty muuta nimeä kuin jäte.

        Jos haluat aloittaa väitelmän siitä, mikä on tieteen tarkoitus, niin ensimmäisenä asiana nousee esiin se, että miksi tämä väitelmä pitää käydä, ja miksi pitää tietää tieteen tarkoitus? Sanoisin että tieteen tarkoitus on tietämisessä, mihin liittyy esim. se että vastakohtana olevasta tietämättömyydestä voi olla käytännönkin haittaa. Sitä ei tulisi sanoa minusta kommunikoinnin tasoksi. Voit verrata siihen, että onko sinusta tieteen tarkoituksen tietäminen ja uudelleen määrittely vain kommunikoinnin tasoa. Kommunikoinnin taso on myös todella käytännöllinen, joten jos parantaa kommunikoinnin tasoa, niin luulisi aina tekevän jotain käytännössä hyödyllistä.

        B:
        "Atomien, molekyylien, elinten ja eliöiden tasot vaikuttavat kaikki rinnakkain samanaikaisesti..."

        Ei ole näyttöä. Esim. voisit aloittaa muutamasta molekyylistä ja sanoa, mitä ne tekevät rinnakkain atomien tekemisen kanssa. Molekyylit voisivat näkyä tosi hyvin laitteilla tämän tarkistamiseksi.

        B:
        "...mutta jos eliön keho on terve ja sillä on jonkintasoista kognitiota (kuten jopa yksisoluisilla tohvelieläimillä on todettu) niin sen eliön molekyylit, atomit ja atomien sisäiset entiteetit liikkuvat sinne minne se kognitio kulloinkin ohjaa eikä mikään fysiikan matemaattinen kaava voi kertoi missä ja miten se tapahtuu ja sama koskee vielä suuremmassa määrin monisoluisia kuten ihmisiä jotka eivät ole mitenkään tarkkaanrajattuja kokonaisuuksia edes elinten ja solujen tasolla vaan säilyttävät kokonaisidentieettinsä aktiivisina toimijoina niin kauan kuin heidän kognitionsa on aktiivisessa yhteydessä heidän kehoonsa eli valveilla ollessa. "

        Mikään kognition selitys ei voi kertoa, missä ja miten tapahtuu tohvelieläimen koko kehon liike. Pidän sitä jotenkin helpompana asiana selittää, vaikka keho on kaiken järjen mukaan monimutkaisempi asia kuin yksi atomi.

        Mikään kognition selitys ei voi kertoa, missä ja miten tapahtuu tohvelielämen sisällä olevan atomin liike.

        Fysiikan kaava pystyy ja on aina jokaisesta kaavaversiosta puhuttaessa pystynyt teoriassa kertomaan tohveliatomin tapahtumat. Niitä kaavoja ei ole koskaan käytetty kyseiseen ongelmaan. Ei siis ole mitään näyttöä mistään asiasta, mitä ne eivät 'voi' tehdä.

        Kognition selitys ei voi teoriassa koskaan kertoa, mitä atomille tulee tapahtumaan. Koska 'kognition selitys' ei ole mitään muuta kuin minun (ei edes B:n) tekemäni sanapari. Sanaparista ei seuraa mitään tietoa.

        Tohvelieläimen kognitio on keksitty todennäköisesti aineen perusteella siten, että otetaan eläin ja pommitetaan sitä aineella, joka on sille aiemmin makroskooppisesti nähtynä biologisesti tuttua. Eläimen ainetta mitataan ja nähdään, että aine eläimessä reagoi pommitukseen aika säännöllisellä tavalla, joka muistuttaa varmaan paljon kuparijohtoja, joissa ei mene kaasua. Sitten toh velieläimestä löytyy pitkän matkan päästä atomi, ja atomi kulkee kohti pommitettua paikkaa. Joku joka ei tunne kaikkea ainetta ja sen liikettä johtojen ja tämän atomin väliltä sanoisi, että välissä oli kognitio, mutta yhtä hyvin atomi on osa samaa virtapiiriä, missä pommitus saa piirin tuomaan toisen piirin osan lähemmäs.

        6

        Itseorganisoitumiseen tarvitaan vähäinen määrä epäjärjestystä???? Näinhän ei ole. Kompleksisissa dynaamisissa järjestelmissä voidaan havaita itseorganisoitumista, vaikka järjestelmä olisi määritelmällisesti kaaoksessa.
        Fysikaalisissa kompleksisissa järjestelmissä esiintyy myös kokonaisuudesta nousevia emergenttejä ilmiöitä. Redusoimattomia.
        Ja ei, kieleksi ei riitä pelkkä matematiikka siinä laajuudessa, mikä meillä on käytettävissä nyt. Tarkat matemaattiset mallit on aina deterministisiä. Todellisuus ei ole.
        Kun pallosalamalle ei ole matemaattista kaavaa, niin olet siis "filosofisesti" sitä mieltä, että se on vain optinen harha? Vaikka se reitillään olisi hetkellisesti vuorovaikuttanut seinän kanssa, jossa muutoksena palojälki. Etkö siis voi ajatella, että se on itseorganisoitunut, ja emergentti ilmiö, joka on todellisuuden osa?
        Tohvelieläimen kognitio? Vai sittenkin vain yksinkertainen atomitason automaattinen reaktio? Ameban genomi on merkittävästi suurempi kuin ihmisen. Se ei ole kovinkaan kompleksinen silti verrokkina.
        Ja matematiikka dynaamisten kompleksisten järjestelmien tulevien tilojen ilmaisijana on lähes hyödytön, koska mitättömän pienet alkuarvojen virheellisyydet saattavat antaa todellisista tapahtumista jopa aivan päinvastaisen tuloksen.


      • Anonyymi
        Anonyymi kirjoitti:

        Itseorganisoitumiseen tarvitaan vähäinen määrä epäjärjestystä???? Näinhän ei ole. Kompleksisissa dynaamisissa järjestelmissä voidaan havaita itseorganisoitumista, vaikka järjestelmä olisi määritelmällisesti kaaoksessa.
        Fysikaalisissa kompleksisissa järjestelmissä esiintyy myös kokonaisuudesta nousevia emergenttejä ilmiöitä. Redusoimattomia.
        Ja ei, kieleksi ei riitä pelkkä matematiikka siinä laajuudessa, mikä meillä on käytettävissä nyt. Tarkat matemaattiset mallit on aina deterministisiä. Todellisuus ei ole.
        Kun pallosalamalle ei ole matemaattista kaavaa, niin olet siis "filosofisesti" sitä mieltä, että se on vain optinen harha? Vaikka se reitillään olisi hetkellisesti vuorovaikuttanut seinän kanssa, jossa muutoksena palojälki. Etkö siis voi ajatella, että se on itseorganisoitunut, ja emergentti ilmiö, joka on todellisuuden osa?
        Tohvelieläimen kognitio? Vai sittenkin vain yksinkertainen atomitason automaattinen reaktio? Ameban genomi on merkittävästi suurempi kuin ihmisen. Se ei ole kovinkaan kompleksinen silti verrokkina.
        Ja matematiikka dynaamisten kompleksisten järjestelmien tulevien tilojen ilmaisijana on lähes hyödytön, koska mitättömän pienet alkuarvojen virheellisyydet saattavat antaa todellisista tapahtumista jopa aivan päinvastaisen tuloksen.

        "Itseorganisoitumiseen tarvitaan vähäinen määrä epäjärjestystä????"

        Lukuunottamatta viimeistä sivun 7 kappaletta, en viittaa nähdäkseni tässä ketjussa koskaan siihen, että minun pitäisi tietää, miten itseorganisoituminen tapahtuisi, ja että pitääkö käsitteitä kuin kaaos liittää siihen ja juuri miten paljon. Monissa osissa tätä ketjua korttien ja numeroiden organisaatioissa tavallista tietoa tästä ei voi juuri käyttää. Yleensä keskusteltu aihe ei edes ole prosessi, millä saavuttaa organisaatiota, vaan organisaatio itse. Puhumme myös, mikä on esim. kortti korttina, mikä puolestaan on sellainen käsite, mitä kaikki muu tarvitsee.

        "Itseorganisoitumiseen tarvitaan vähäinen määrä epäjärjestystä???? Näinhän ei ole. Kompleksisissa dynaamisissa järjestelmissä voidaan havaita itseorganisoitumista, vaikka järjestelmä olisi määritelmällisesti kaaoksessa."

        Jos et tarvitse vähäistä määrää epäjärjestystä, niin koeta havaita itseorganisoitumista (a) kompleksisessa järjestelmässä, joka ei ole määritelmällisesti kaaoksessa tai (b) dynaamisessa järjestelmässä, joka ei ole edes kompleksinen.

        Aiheesta on jo pitkä kirjoitus tässä ketjussa:
        https://keskustelu.suomi24.fi/t/17988944/filosofian-uusi-tuleminen#comment-123440153
        eli sivulla 2 viimeisessä kappaleessa on esitetty kyseinen väite.

        Toisen ketjun sivulla 7 on tarkempi tapaus siitä, miten linkitetyssä kuvassa oleva idea yleensä tulee, ja miksi kuva ei ole selityksenä paljon minkään arvoinen.

        "Fysikaalisissa kompleksisissa järjestelmissä esiintyy myös kokonaisuudesta nousevia emergenttejä ilmiöitä. Redusoimattomia.
        Ja ei, kieleksi ei riitä pelkkä matematiikka siinä laajuudessa, mikä meillä on käytettävissä nyt. Tarkat matemaattiset mallit on aina deterministisiä. Todellisuus ei ole.
        Kun pallosalamalle ei ole matemaattista kaavaa, niin olet siis "filosofisesti" sitä mieltä, että se on vain optinen harha?"

        Jos et ole käyttänyt matematiikkaa tai yhtään mitään muutakaan osoittamaan, että jostain nousee jotain vahvasti emergenttiä, niin 'filosofia' on, että pidän tätä epäilyksen alaisena. Ja yhtä hyvin on mahdollista, että kaikki on selitettävissä matematiikalla, ja voi olla myös, että se on jo selitettävissä matematiikalla, jota jo nyt on. Sitä matematiikkaa ei vain ole kirjoitettu kokonaan läpi jokaista vaadittua elementtiä varten ja suoritettu tietokoneella. Periaatteessa todellisuudessa voi olla jotain, mihin nykyinen matematiikka ei riitä, mutta se mikä tämä asia on, ei ole välttämättä vielä edes tullut kenenkään mieleen. Tai että se olisi juuri esim. kaikkien tietämä tietoisuus ja huom. koko tämä käsite kokonaisuudessaan (vaikka mitään sovittuja kokonaisuuksia ei edes ole annettu). Sen sijaan olisi niin, että tietoisuudesta pystyy tämän hetken fysiikalla mallintamaan esim. tietyn verran, ja eilisen fysiikalla vähän sitä vähemmän.

        Salamille on samat kaavat kuin plasmalle yleensä. Tässä joku käyttää fluidi-yhtälöitä salamaan. Ja seinän kanssa:
        https://arxiv.org/abs/1906.08521

        "Vaikka se reitillään olisi hetkellisesti vuorovaikuttanut seinän kanssa, jossa muutoksena palojälki. Etkö siis voi ajatella, että se on itseorganisoitunut..."

        En, koska itseorganisoitumistieteessä on kyse jostain muusta. Seinien potentiaalit pakottavat salaman elektonit tässä johonkin. Todennäköisesti seinän kiderakenne hajoaa enemmän kuin rakentuu. Tämä rakentuminen on puolestaan tehty syöttämällä lisäenergiaa kilometrien päästä eli kaukaa seinän tyypillisestä itsestä.

        "Ameban genomi on merkittävästi suurempi kuin ihmisen. Se ei ole kovinkaan kompleksinen silti verrokkina."

        Jos tämä tiedetään, on käytetty matematiikkaa ja on käytetty objekteja, joihin genomi on jaettu. Sillä samalla mallilla voitaisiin myös tehdä jotain muuta kuin vertailla.

        Seuraavassa sanotaan kuin matematiikka olisi hyödytön ilman ennustamista, mutta minä sanoin vahingossa, että matematiikka on hyödytöntä, jos sillä vertailee. Jossain tässä välissä matematiikasta on ollut paljon eri hyötyjä.

        "Ja matematiikka dynaamisten kompleksisten järjestelmien tulevien tilojen ilmaisijana on lähes hyödytön, koska mitättömän pienet alkuarvojen virheellisyydet saattavat antaa todellisista tapahtumista jopa aivan päinvastaisen tuloksen."

        Tästä pääteltäisiin, että ameban tietoisuus on lähes hyödytön. Sillä ei ole mitään ennustettavaa asiaa tietää mitään minkään perusteella, jos se alkaa aina tietää toisia asioita.

        Sanoit että se toinen osa aivan samassa tilanteessa on, että voi havaita itseorganisoitumista? Tämä liittyy myös tulevaisuuteen amebassa?


      • Anonyymi
        Anonyymi kirjoitti:

        "Itseorganisoitumiseen tarvitaan vähäinen määrä epäjärjestystä????"

        Lukuunottamatta viimeistä sivun 7 kappaletta, en viittaa nähdäkseni tässä ketjussa koskaan siihen, että minun pitäisi tietää, miten itseorganisoituminen tapahtuisi, ja että pitääkö käsitteitä kuin kaaos liittää siihen ja juuri miten paljon. Monissa osissa tätä ketjua korttien ja numeroiden organisaatioissa tavallista tietoa tästä ei voi juuri käyttää. Yleensä keskusteltu aihe ei edes ole prosessi, millä saavuttaa organisaatiota, vaan organisaatio itse. Puhumme myös, mikä on esim. kortti korttina, mikä puolestaan on sellainen käsite, mitä kaikki muu tarvitsee.

        "Itseorganisoitumiseen tarvitaan vähäinen määrä epäjärjestystä???? Näinhän ei ole. Kompleksisissa dynaamisissa järjestelmissä voidaan havaita itseorganisoitumista, vaikka järjestelmä olisi määritelmällisesti kaaoksessa."

        Jos et tarvitse vähäistä määrää epäjärjestystä, niin koeta havaita itseorganisoitumista (a) kompleksisessa järjestelmässä, joka ei ole määritelmällisesti kaaoksessa tai (b) dynaamisessa järjestelmässä, joka ei ole edes kompleksinen.

        Aiheesta on jo pitkä kirjoitus tässä ketjussa:
        https://keskustelu.suomi24.fi/t/17988944/filosofian-uusi-tuleminen#comment-123440153
        eli sivulla 2 viimeisessä kappaleessa on esitetty kyseinen väite.

        Toisen ketjun sivulla 7 on tarkempi tapaus siitä, miten linkitetyssä kuvassa oleva idea yleensä tulee, ja miksi kuva ei ole selityksenä paljon minkään arvoinen.

        "Fysikaalisissa kompleksisissa järjestelmissä esiintyy myös kokonaisuudesta nousevia emergenttejä ilmiöitä. Redusoimattomia.
        Ja ei, kieleksi ei riitä pelkkä matematiikka siinä laajuudessa, mikä meillä on käytettävissä nyt. Tarkat matemaattiset mallit on aina deterministisiä. Todellisuus ei ole.
        Kun pallosalamalle ei ole matemaattista kaavaa, niin olet siis "filosofisesti" sitä mieltä, että se on vain optinen harha?"

        Jos et ole käyttänyt matematiikkaa tai yhtään mitään muutakaan osoittamaan, että jostain nousee jotain vahvasti emergenttiä, niin 'filosofia' on, että pidän tätä epäilyksen alaisena. Ja yhtä hyvin on mahdollista, että kaikki on selitettävissä matematiikalla, ja voi olla myös, että se on jo selitettävissä matematiikalla, jota jo nyt on. Sitä matematiikkaa ei vain ole kirjoitettu kokonaan läpi jokaista vaadittua elementtiä varten ja suoritettu tietokoneella. Periaatteessa todellisuudessa voi olla jotain, mihin nykyinen matematiikka ei riitä, mutta se mikä tämä asia on, ei ole välttämättä vielä edes tullut kenenkään mieleen. Tai että se olisi juuri esim. kaikkien tietämä tietoisuus ja huom. koko tämä käsite kokonaisuudessaan (vaikka mitään sovittuja kokonaisuuksia ei edes ole annettu). Sen sijaan olisi niin, että tietoisuudesta pystyy tämän hetken fysiikalla mallintamaan esim. tietyn verran, ja eilisen fysiikalla vähän sitä vähemmän.

        Salamille on samat kaavat kuin plasmalle yleensä. Tässä joku käyttää fluidi-yhtälöitä salamaan. Ja seinän kanssa:
        https://arxiv.org/abs/1906.08521

        "Vaikka se reitillään olisi hetkellisesti vuorovaikuttanut seinän kanssa, jossa muutoksena palojälki. Etkö siis voi ajatella, että se on itseorganisoitunut..."

        En, koska itseorganisoitumistieteessä on kyse jostain muusta. Seinien potentiaalit pakottavat salaman elektonit tässä johonkin. Todennäköisesti seinän kiderakenne hajoaa enemmän kuin rakentuu. Tämä rakentuminen on puolestaan tehty syöttämällä lisäenergiaa kilometrien päästä eli kaukaa seinän tyypillisestä itsestä.

        "Ameban genomi on merkittävästi suurempi kuin ihmisen. Se ei ole kovinkaan kompleksinen silti verrokkina."

        Jos tämä tiedetään, on käytetty matematiikkaa ja on käytetty objekteja, joihin genomi on jaettu. Sillä samalla mallilla voitaisiin myös tehdä jotain muuta kuin vertailla.

        Seuraavassa sanotaan kuin matematiikka olisi hyödytön ilman ennustamista, mutta minä sanoin vahingossa, että matematiikka on hyödytöntä, jos sillä vertailee. Jossain tässä välissä matematiikasta on ollut paljon eri hyötyjä.

        "Ja matematiikka dynaamisten kompleksisten järjestelmien tulevien tilojen ilmaisijana on lähes hyödytön, koska mitättömän pienet alkuarvojen virheellisyydet saattavat antaa todellisista tapahtumista jopa aivan päinvastaisen tuloksen."

        Tästä pääteltäisiin, että ameban tietoisuus on lähes hyödytön. Sillä ei ole mitään ennustettavaa asiaa tietää mitään minkään perusteella, jos se alkaa aina tietää toisia asioita.

        Sanoit että se toinen osa aivan samassa tilanteessa on, että voi havaita itseorganisoitumista? Tämä liittyy myös tulevaisuuteen amebassa?

        En moossään kohdassa antanut ymmärtää, että amebat on tietoisia, joten johtopäätöksesi kysymyksesi muodostamiseen on mieletön. Älä pidä matematiikkaa erehtymättömänä, vaikka se olisikin "nerojen" käsialaa. Koska edelleen todellisuutta on se, että matemaatikot mallinnoksissaan "pyöristää" alkuarvolukuja. Approksimoi, jolloin vain osa tiedoista on kokonaisuudessa mukana, ja pois on tiputettu tunnistamattomat, tai yhtälöihin muuten epävarmuudeltaan määrittelemätöntä vaihtelua tuovat tekijät, tai sitten vain pyöristetty, jolloin loppuarvot on arpaonnitasoa. Niin paljon kun matematiikkaa arvostankin, niin realistinen täytyy silti olla. Matemaatikot kehittelevät symboliärjestelmiä, joista tiede saattaa ottaa kopin vasta vuosikymmenien päästä. Jos ottaa, ja sitten nostaa vajavaisuudet jalustalle.

        Emergentiksi voisi kutsua vaikka tahdonvoimaa, joka sekin nousee emergentiksi todetusta ilmiöstä nimeltä tietoisuus. Se ei ole uskomusta, vaan mitattavissa ja havainnoitavissa olevaa mielen vaikutusta monimutkaiseen dynaamiseen järjestelmään, jota voidaan kutsua ihmiskehoksi. "Mieli" (tahdonalainen ja tietoinen), voi siis vaikuttaa normisti tahdosta riippumattomiin biologisiin prosesseihin, ja mikrotason ilmiöihin.

        Paljon puhuttua, mutta paljon tutkittuakin. "Nocebo-" ja "placebo"-ilmiöt ehkä tutkituimpia. Molemmista kokeellista empiiristä näyttöä paljon. Ja aivokuvantamisen kehittyminen laajensi tutkimusta suunnattomasti eteenpäin.
        Ylätaso-orientoituneen ja "deterministisen" fysiikan vastaisesti, tai käänteisesti voi makrotasolla vaikuttaa jopa tahdosta riippumattomiin toimintoihin. Rauhoittaa sykkeen ja hengityksen, minimoida kivun jne.
        Ja ihmisen kohdalla (yksilökohtaisesti) makrotasolla on mahdollisuus hallita alatasojaan jonkin verran.

        Tietoisuus ei todellakaaan ole hyödytön. Sitä ei saman tason tiedostamatonkaan ole.


    • Luonnonlaitkin pitäisi varmaan redusoida johonkin.

    • Anonyymi

      Nykykäsitys taitaa olla se että kvanttikietoutumisen avulla ei voi lähettää informaatiota nopeammin kuin valonnopeus koska syy ja seuraus suhteet menisivät pieleen.

      "Kvanttikietoutuminen on kvanttimekaaninen ilmiö, jossa kahden tai useamman kappaleen kvanttitilat on kuvattava suhteessa toisiinsa, vaikka kappaleet olisivatkin erillään avaruudessa. Tämä tarkoittaa sitä, että kappaleiden ominaisuudet, kuten paikka, liike, pyöriminen ja energia, ovat riippuvaisia toisistaan ja vaikuttavat toisiinsa etäisyydestä riippumatta."


      ...
      ..

    Ketjusta on poistettu 1 sääntöjenvastaista viestiä.

    Luetuimmat keskustelut

    1. Katso: Ohhoh! Miina Äkkijyrkkä sai käskyn lähteä pois Farmi-kuvauksista -Kommentoi asiaa: "En ole.."

      Tämä oli shokkiyllätys. Oliko tässä kyse tosiaan siitä, että Äkkijyrkkä sanoi asioita suoraan vai mistä.... Tsemppiä, Mi
      Tv-sarjat
      140
      6123
    2. Voi kun mies rapsuttaisit mua sieltä

      Saisit myös sormiisi ihanan tuoksukasta rakkauden mahlaa.👄
      Ikävä
      33
      3705
    3. Poliisiauto Omasp:n edessä parkissa

      Poliisiauto oli parkissa monta tuntia Seinäjoen konttorin edessä tänään. Haettiinko joku tai jotain pankista tutkittavak
      Seinäjoki
      25
      1955
    4. Haluan jutella kanssasi Nainen

      Olisiko jo aika tavata ja avata tunteemme...On niin paljon asioita joihin molemmat ehkä haluaisimme saada vastaukset...O
      Ikävä
      18
      1939
    5. Onko mies niin,

      että sinulle ei riitä yksi nainen? Minulle suhde tarkoittaa sitoutumista, tosin eihän se vankila saa olla kummallekaan.
      Tunteet
      25
      1768
    6. Voitasko leikkiä jotain tunnisteleikkiä?

      Tietäisi ketä täällä käy kaipaamassa.. kerro jotain mikä liittyy sinuun ja häneen eikä muut tiedä. Vastaan itsekin kohta
      Ikävä
      76
      1619
    7. Tietysti jokainen ansaitsee

      Hän varmasti ansaitsee vain parasta ja sopivinta tietenkin, suon sen onnen hänelle enemmän kuin mielelläni. Aika on nyt
      Ikävä
      16
      1542
    8. Armi Aavikko Malmin hautausmaa

      Haudattiinko Armi arkussa Malmin hautausmaalle vai tuhkattiinko hänet? Kuka tietää asiasta oikein?
      Kotimaiset julkkisjuorut
      14
      1445
    9. Haluisin suudella ja huokailla

      ja purra kaulaasi ja rakastella sinua. Haluisin puristella rintojasi ja pakaroitasi. Ei sinulla taida olla kuitenkaan ni
      Ikävä
      18
      1438
    10. Miksi näin?

      Miksi vihervassut haluaa maahan porukkaa jonka pyhä kirja kieltää sopeutumisen vääräuskoisten keskuuteen? Näin kotoutumi
      Maailman menoa
      27
      1355
    Aihe